Tag Archives: TPHCM

Đề và đáp án thi vào lớp 10 Chuyên Toán TPHCM năm 2022

Bài 1. (1,0 diểm)
Cho $x, y$ là hai số thực thỏa mãn $x y+\sqrt{\left(1+x^2\right)\left(1+y^2\right)}=1$.
Tính giá trị của biểu thức $M=\left(x+\sqrt{1+y^2}\right)\left(y+\sqrt{1+x^2}\right)$.
Bài 2. (2,5 diểm)
a) Giải phương trình $\sqrt{x+4}+|x|=x^2-x-4$.
Bài 3. (1,5 diểm)
Cho hình vuông $A B C D$ Trên các cạnh $B C$ và $C D$ lần lượt lấy các điểm $M$ và $N$ sao cho $\angle M A N=45^{\circ}$.
a) Chứng minh $M N$ tiếp xúc với dường tròn tâm $A$ bán kính $A B$.
b) Kẻ $M P$ song song với $A N$ ( $P$ thuộc đoạn $A B)$ và kẻ $N Q$ song song với $A M(Q$ thuộc đoạn $A D)$. Chứng minh $A P=A Q$.
Bài 4. (2,0 diểm)
Cho ba số thực dương $a, b, c$ thỏa $a+b+c=3$.
a) Chứng minh rằng $a b+b c+c a \leq 3$.
b) Tìm giá trị nhỏ nhất của biểu thức $P=\frac{a}{b^2+1}+\frac{b}{c^2+1}+\frac{c}{a^2+1}$.
Bài 5. (2,0 diểm)
Cho tam giác $A B C$ nhọn $(A B<A C)$ có các đường cao $A D, B E, C F$ cắt nhau tại $H$. Đường thẳng $E F$ cắt đường thẳng $B C$ tại $I$. Đường thẳng qua $A$ vuông góc với $I H$ tại $K$ và cắt $B C$ tại $M$.
a) Chứng minh tứ giác $I F K C$ nội tiếp và $\frac{B I}{B D}=\frac{C I}{C D}$.
b) Chứng minh $M$ là trung diểm của $B C$.

Bài 6. (1,0 diểm)
Số nguyên dương $n$ được gọi là “số tốt” nếu $n+1$ và $8 n+1$ dều là các số chính phương.
a) Hãy chỉ ra ví dụ ba “số tốt” lần lượt có 1, 2, 3 chữ số.
b) Tìm các số nguyên $k$ thỏa mãn $|k| \leq 10$ và $4 n+k$ là hợp số với mọi $n$ là “số tốt”.

Đáp án được thực hiện vởi Star Education

Bài 1.

Điều kiện: $x y \leq 1$. Biến đổi giả thiết
$$
\sqrt{\left(1+x^2\right)\left(1+y^2\right)}=1-x y \Leftrightarrow\left(1+x^2\right)\left(1+y^2\right)=(1-x y)^2 \Leftrightarrow(x+y)^2=0 \Leftrightarrow y=-x .
$$
Thay vào biểu thức $M$ ta được
$$
\begin{aligned}
M & =\left(x+\sqrt{1+y^2}\right)\left(y+\sqrt{1+x^2}\right) \
& =\left(x+\sqrt{1+x^2}\right)\left(-x+\sqrt{1+x^2}\right) \
& =\left(\sqrt{1+x^2}\right)^2-x^2=1
\end{aligned}
$$

Bài 2.

a)

Lời giải:
a) Điều kiện: $\left\{\begin{array}{l}x+4 \geq 0 \\\\ x^2-x-4 \geq 0\end{array} \right.$

$\Leftrightarrow\left[\begin{array}{l}-4 \leq x \leq \frac{1-\sqrt{17}}{2} \\\\ x \geq \frac{1+\sqrt{17}}{2}\end{array}\right.$
Phương trình đã cho tương đương
$$
x^2-\sqrt{x+4}-|x|-(x+4)=0 \Leftrightarrow(|x|+\sqrt{x+4})(|x|-\sqrt{x+4}-1)=0 \Leftrightarrow|x|-1=\sqrt{x+4}
$$

  • Nếu $x \geq 0,(1) \Rightarrow x-1=\sqrt{x+4}$
    $$
    \Rightarrow x^2-2 x+1=x+4 \Leftrightarrow x^2-3 x-3=0 \Leftrightarrow\left[\begin{array}{l}
    x=\frac{3+\sqrt{21}}{2} \text { (Nhận) } \\\\
    x=\frac{3-\sqrt{21}}{2} \text { (Loại) }
    \end{array}\right.
    $$
  • Nếu $x<0,(1) \Rightarrow-x-1=\sqrt{x+4}$
    $$
    \Rightarrow x^2+2 x+1=x+4 \Leftrightarrow x^2+x-3=0 \Leftrightarrow\left[\begin{array}{l}
    x=\frac{-1+\sqrt{13}}{2} \text { (Loại) } \\\\
    x=\frac{-1-\sqrt{13}}{2} \text { (Nhận) }
    \end{array} .\right.
    $$
    Thử lại, ta được $x=\frac{3+\sqrt{21}}{2}$ và $x=\frac{-1-\sqrt{13}}{2}$ là các nghiệm của phương trình đã cho.

b) Điều kiện: $(x+y)(y+z)(z+x) \neq 0$. Hệ dã cho tương dương
$$
\left\{\begin{array} { l }
{ \frac { x } { y + z } + 1 = 2 x } \\\\
{ \frac { y } { z + x } + 1 = 3 y } \\\\
{ \frac { z } { x + y } + 1 = 5 z }
\end{array} \Leftrightarrow \left\{\begin{array} { l }
{ \frac { x + y + z } { y + z } = 2 x } \\\\
{ \frac { x + y + z } { z + x } = 3 y } \\\\
{ \frac { x + y + z } { x + y } = 5 z }
\end{array} \Leftrightarrow \left\{\begin{array}{l}
x+y+z=2 x(y+z) \\\\
x+y+z=3 y(z+x) \\\\
x+y+z=5 z(x+y)
\end{array}\right.\right.\right.
$$
Dễ thấy $x y z \neq 0$. Từ trên suy ra
$$
2 x(y+z)=3 y(z+x)=5 z(x+y) \Leftrightarrow 2\left(\frac{1}{y}+\frac{1}{z}\right)=3\left(\frac{1}{z}+\frac{1}{x}\right)=5\left(\frac{1}{x}+\frac{1}{y}\right) .
$$
Ta tính được $\frac{1}{z}=\frac{19}{x}, \frac{1}{y}=\frac{11}{x} \Rightarrow x=11 y=19 z$. Thay lại vào phương trình $(*)$ ta dược
$$
x+\frac{x}{11}+\frac{x}{19}=2 x\left(\frac{x}{11}+\frac{x}{19}\right) \Leftrightarrow 1+\frac{1}{11}+\frac{1}{19}=2\left(\frac{x}{11}+\frac{x}{19}\right) \Leftrightarrow x=\frac{239}{60} .
$$
Suy ra $y=\frac{239}{660}, z=\frac{239}{1140}$.
Vậy nghiệm duy nhất của hệ là $(x, y, z)=\left(\frac{239}{60}, \frac{239}{660}, \frac{239}{1140}\right)$.

Bài 3.

a) Trên tia đối của tia $D C$ lấy $F$ sao cho $D F=B M$.
Xét $\triangle A D F$ và $\triangle A B M$ có $A D=A B, \angle A D F=\angle A B M=90^{\circ}$ và $D F=B M$.
Do đó $\triangle A D F=\triangle A B M(\mathrm{c}-\mathrm{g}-\mathrm{c})$
$\Rightarrow \angle D A F=\angle B A M$ và $A F=A M$.
Suy ra $\angle D A F+\angle D A N=\angle B A M+\angle D A N=90^{\circ}-45^{\circ}=45^{\circ}$.
$\Rightarrow \angle N A F=45^{\circ}=\angle N A M$, mà $A F=A M$ nên $\triangle N A F=\triangle N A M$. (c-g-c)
Kẻ $A E \perp M N(E \in M N) \Rightarrow A E=A D=A B \Rightarrow M N$ tiếp xúc với $(A, A B)$.
b) Ta có: $\triangle N A F=\triangle N A M \Rightarrow \angle A N F=\angle A N M$, mà $\angle A N F=\angle N A P($ do $D C | A B)$, dẫn đến $\angle A N M=\angle N A P$.

Từ $A N | M P \Rightarrow A P M N$ là hình thang, kết hợp với $\angle A N M=\angle N A P$, ta được $A P M N$ là hình thang cân.
Do đó $A P=M N$, tương tự ta cũng có $A Q=M N$, dẫn dến $A P=A Q$.

Bài 4.

a)

a) Ta có $a^2+b^2 \geq 2 a b, b^2+c^2 \geq 2 b c, c^2+a^2 \geq 2 c a$ nên
$$
2\left(a^2+b^2+c^2\right) \geq 2(a b+b c+c a) \Leftrightarrow a^2+b^2+c^2 \geq a b+b c+c a .
$$
Khi đó
$$
\begin{aligned}
9=(a+b+c)^2 & =a^2+b^2+c^2+2 a b+2 b c+2 c a \
& \geq a b+b c+c a+2(a b+b c+c a)=3(a b+b c+c a)
\end{aligned}
$$
Do đó $a b+b c+c a \leq 3$.
Dấu “=” xảy ra khi và chỉ khi $a=b=c=1$.

b)

b) Ta có
$$
\begin{aligned}
& \frac{a}{b^2+1}-a=\frac{-a b^2}{b^2+1} \geq-\frac{a b^2}{2 b}=-\frac{a b}{2} \
& \frac{b}{c^2+1}-b=\frac{-b c^2}{c^2+1} \geq-\frac{b c^2}{2 c}=-\frac{b c}{2} \
& \frac{c}{a^2+1}-c=\frac{-c a^2}{a^2+1} \geq-\frac{c a^2}{2 a}=-\frac{c a}{2}
\end{aligned}
$$
Do đó
$$
\begin{aligned}
& \frac{a}{b^2+1}+\frac{b}{c^2+1}+\frac{c^2}{a^2+1}-(a+b+c) \geq-\frac{a b+b c+c a}{2} \geq-\frac{3}{2} \
\Rightarrow & \frac{a}{b^2+1}+\frac{b}{c^2+1}+\frac{c}{a^2+1} \geq-\frac{3}{2}+a+b+c=\frac{3}{2}
\end{aligned}
$$
Vậy giá trị nhỏ nhất của $P$ là $\frac{3}{2}$, dấu “=” xảy ra khi và chỉ khi $a=b=c=1$.

Bài 5.

Vẽ dường tròn $(O)$ ngoại tiếp $\triangle A B C$
a) Ta có: Các tứ giác $A F D C, A K D I, B F E C, A F H E$ nội tiếp.
$\Rightarrow H F \cdot H C=H D \cdot H A=H K . H I \Rightarrow I F K C$ nội tiếp.
Mặt khác: $\widehat{I F B}=\widehat{A C B}=\widehat{B F D}$ (do các tứ giác $B F E C, A F D C$ nội tiếp)
$\Rightarrow F B$ là phân giác $\widehat{I F D}$.
Mà $F B \perp F C$ nên $F B$ là phân giác trong, $F C$ là phân giác ngoài $\triangle I F D$
$$
\Rightarrow \frac{B I}{B D}=\frac{C I}{C D}
$$
b) Gọi $S$ là giao điểm thứ hai của $I A$ và đường tròn ngoại tiếp $O$.
Ta chứng minh được $I F . I E=I B . I C=I S . I A$
$\Rightarrow A S F E$ nội tiếp hay 5 điểm $A, S, F, H, E$ cùng thuộc đường tròn đường kính $A H$
$\Rightarrow \widehat{A S H}=\widehat{A F H}=90^{\circ}$
Mặt khác do: $I K \perp A M, A D \perp I M$ nên $H$ là trực tâm $\triangle A I M \Rightarrow M H \perp A I$.
Từ đó, ta có: $S, H, M$ thẳng hàng.
Vẽ đường kính $A Q$ của đường tròn ngoại tiếp $\triangle A B C$.
Ta có $\widehat{A S Q}=90^{\circ}$ nên $S, H, M, Q$ thẳng hàng
Xét tứ giác $B H C Q$ có: $B H / / C Q$ (cùng $\perp A C)$ và $C H / / B Q($ cùng $\perp A B)$
Nên $B H C Q$ là hình bình hành nghĩa là có $M$ là trung điểm $B C$.

Bài 6.

Lời giải:
a) Ví dụ: $3\left(3+1=2^2\right.$ và $\left.8 \cdot 3+1=5^2\right), 15\left(15+1=4^2\right.$ và $\left.8 \cdot 15+1=11^2\right)$ và 120 $\left(120+1=11^2\right.$ và $\left.8 \cdot 120+1=31^2\right)$.
b) Nhận xét $a^2 \equiv 0,1(\bmod 3)$ với mọi $a \in \mathbb{N}$.
Đặt $n+1=x^2$ và $8 n+1=y^2(x, y \in \mathbb{N})$.

  • Nếu $n \equiv 1(\bmod 3)$ thì $x^2=n+1 \equiv 2(\bmod 3)$, vô lí.
  • Nếu $n \equiv 2(\bmod 3)$ thì $y^2=8 n+1 \equiv 17 \equiv 2(\bmod 3)$, vô lí.
    Vậy $n \equiv 0(\bmod 3)$ hay $n$ chia hết cho 3 .
    Nếu $k=1,5,7,-5,-7$ thì với $n=3$ (là số tốt), $4 n+k$ nhận các giá trị $13,17,19,7,5$ là các số nguyên tố. (Loại)
    Nếu $k=-1$, với $n=15$ (là số tốt) thì $4 n+k=59$ là số nguyên tố. (Loại)
    Nếu $k=-10$, với $n=3$ thì $4 n+k=2$ là số nguyên tố. (Loại)
    Nếu $k=-9$, với $n=3$ thì $4 n+k=3$ là số nguyên tố. (Loại)
    Nếu $k \geq-8, k$ chẵn hoặc $k$ chia hết cho 3 thì $4 n+k \geq 4 \cdot 3-8=4$ và $4 n+k$ có ước là 2 hoặc 3 , do đó $4 n+k$ là hợp số.
    Vậy các giá trị cần tìm của $k$ là
    $$
    k \in{-8,-6,-4,-3,-2,0,2,3,4,6,8,9,10} .
    $$

ĐỀ THI VÀO CHUYÊN TOÁN LỚP 10 TP.HCM 2012

Bài 1. Giải phương trình:

$\sqrt{8 x+1}+\sqrt{46-10 x}=-x^{3}+5 x^{2}+4 x+1$

Bài 2. Cho đa thức $f(x)=a x^{3}+b x^{2}+c x+d$ với $a$ là số nguyên dương, biết $f(5)-$ $f(4)=$ 2012. Chứng minh rằng: $f(7)-f(2)$ là hợp số.

Bài 3. Cho ba số dương $a, b, c$ thỏa $a+b+c=1$. Tìm giá trị nhỏ nhất của biểu thức:

$A=14\left(a^{2}+b^{2}+c^{2}\right)+\frac{a b+b c+c a}{a^{2} b+b^{2} c+c^{2} a}$

Bài 4. Cho tứ giác $A B C D$ nội tiếp đường tròn $(O, R)$ có $A C$ vuông góc với $B D$ tại $H$. Trên cạnh $A B$ lấy điểm $M$ sao cho $A B=3 A M$. Trên cạnh $H C$ lấy trung điểm $N$. Chứng minh rằng $M H$ vuông góc với $D N$.

Bài 5. Cho đường tròn tâm $O$ và đường tròn tâm $I$ cắt nhau tại hai điểm $A$ và $B(O$ và $I$ nằm khác phía đối với đường thẳng $A B), I B$ cắt $(O)$ tại $E, O B$ cắt $(I)$ tại $F$. Qua $B$ vẽ đường thẳng $M N$ song song với $E F(M$ thuộc $(O), N$ thuộc $(I)$ ).

(a) Chứng minh rằng $O A I E$ nội tiếp.

(b) Chứng minh rằng: $A E+A F=M N$.

Bài 6. Trên mặt phẳng cho 2013 điểm tùy ý sao cho trong ba điểm bất kì thì tồn tại 2 điểm mà khoảng cách giữa hai điểm đó luôn bé hơn 1. Chứng minh rằng tồn tại một đường tròn có bán kính bằng 1 chứa ít nhất 1007 điểm (kế cả biên).

 

LỜI GIẢI

Bài 1. Giải phương trình:

$\sqrt{8 x+1}+\sqrt{46-10 x}=-x^{3}+5 x^{2}+4 x+1$

Lời giải. $\sqrt{8 x+1}+\sqrt{46-10 x}=-x^{3}+5 x^{2}+4 x+1$

ĐKХĐ: $\frac{-1}{8} \leq x \leq \frac{23}{5}$

Sử dụng lượng liên hợp, phương trình ban đầu tương đương với:

$\sqrt{8 x+1}-3+\sqrt{46-10 x}-6+x^{3}-x^{2}-4 x^{2}+4 x-8 x+8=0$

$\Leftrightarrow(x-1)\left(\frac{8}{\sqrt{8 x+1}+3}-\frac{10}{\sqrt{46-10 x}+6}+x^{2}-4 x-8\right)=0$

Từ đó ta có phương trình có một nghiệm là $x=1$. Xét biểu thức:

$\frac{8}{\sqrt{8 x+1}+3}-\frac{10}{\sqrt{46-10 x}+6}+x^{2}-4 x-8=0$

Từ điều kiện ta có:

$-1<x<5 \Leftrightarrow(x+1)(x-5)<0 \Leftrightarrow x^{2}-4 x-5<0$

Lại có: $\frac{8}{\sqrt{8 x+1}+3} \leq \frac{8}{3}<\frac{9}{3}=3 \Leftrightarrow \frac{8}{\sqrt{8 x+1}+3}-3<0$ Từ đó ta có:

$\frac{8}{\sqrt{8 x+1}+3}-\frac{10}{\sqrt{46-10 x}+6}+x^{2}-4 x-8<0$

Vậy phương trình đã cho có nghiệm duy nhất là: $x=1$

Bài 2. Cho đa thức $f(x)=a x^{3}+b x^{2}+c x+d$ với $a$ là số nguyên dương, biết $f(5)-$ $f(4)=2012$. Chứng minh rằng: $f(7)-f(2)$ là hợp số.

Lời giải. Ta có: $f(x)=a x^{3}+b x^{2}+c x+d$

Từ đó ta tính được: $f(5)=125 a+25 b+5 c+d, f(4)=64 a+16 b+4 c+d$

Vậy: $f(5)-f(4)=61 a+9 b+c=2012, f(7)=343 a+49 b+7 c+d, f(2)=8 a+4 b+$ $2 c+d$

Vậy: $f(7)-f(2)=335 a+45 b+5 c=5(67 a+9 b+c)=30 a+5(61 a+9 b+c)=30 a+$ 10060

Từ đó ta có: $f(7)-f(2)$ là hợp số vì $a$ là số nguyên dương và nó chia hết cho $2,5,10$.

Bài 3. Cho ba số dương $a, b, c$ thỏa $a+b+c=1$. Tìm giá trị nhỏ nhât của biểu thức:

$A=14\left(a^{2}+b^{2}+c^{2}\right)+\frac{a b+b c+c a}{a^{2} b+b^{2} c+c^{2} a}$

Lời giải.

Cách 1:

$\left(a^{2}+b^{2}+c^{2}\right)(a+b+c)=a^{3}+b^{3}+c^{3}+\left(a^{2} b+b^{2} c+c^{2} a\right)+\left(b^{2} a+a^{2} c+c^{2} b\right) $

$\left(a^{2}+b^{2}+c^{2}\right)(a+b+c)=\left(a^{3}+a b^{2}\right)+\left(b^{3}+b c^{2}\right)+\left(c^{3}+c a^{2}\right)+\left(a^{2} b+b^{2} c+c^{2} a\right)$

Áp dụng bất đẳng thức Cauchy và do $a+b+c=1$, ta có:

$\left(a^{2}+b^{2}+c^{2}\right) \geq 2 a^{2} b+2 b^{2} c+2 c^{2} a+\left(a^{2} b+b^{2} c+c^{2} a\right)=3\left(a^{2} b+b^{2} c+c^{2} a\right)$

Mặt khác: $a b+b c+c a=\frac{1-\left(a^{2}+b^{2}+c^{2}\right)}{2}$

Từ đó ta có: $F \geq 14\left(a^{2}+b^{2}+c^{2}\right)+\frac{3-3\left(a^{2}+b^{2}+c^{2}\right)}{2\left(a^{2}+b^{2}+c^{2}\right)}$

Hay: $F \geq 14\left(a^{2}+b^{2}+c^{2}\right)+\frac{3}{2\left(a^{2}+b^{2}+c^{2}\right)}-\frac{3}{2}$

Áp dụng bất đẳng thức Cauchy, ta có:

$27\left(a^{2}+b^{2}+c^{2}\right)+\frac{3}{\left(a^{2}+b^{2}+c^{2}\right)} \geq 2 \sqrt{27\left(a^{2}+b^{2}+c^{2}\right) \cdot \frac{3}{\left(a^{2}+b^{2}+c^{2}\right)}}=18 $

$a^{2}+b^{2}+c^{2} \geq \frac{1}{3}(a+b+c)^{2}=\frac{1}{3}$

Vậy: $28\left(a^{2}+b^{2}+c^{2}\right)+\frac{3}{\left(a^{2}+b^{2}+c^{2}\right)} \geq 18+\frac{1}{3}=\frac{55}{3}$

Từ đó ta có: $F \geq \frac{55}{6}-\frac{3}{2}=\frac{23}{3}$

Đẳng thức xảy ra khi: $a=b=c=\frac{1}{3}$

Cách 2:

Do $a, b, c$ dương và $a+b+c=1$ nên ta có:

$(1-c)^{2}=(a+b)^{2} \geq 4 a b \Leftrightarrow 1-2 c+c^{2} \geq 4 a b \Leftrightarrow a-2 a c+a c^{2} \geq 4 a^{2} b $

$(1-a)^{2}=(b+c)^{2} \geq 4 b c \Leftrightarrow 1-2 a+a^{2} \geq 4 b c \Leftrightarrow b-2 a b+a^{2} b \geq 4 b^{2} c $

$(1-b)^{2}=(c+a)^{2} \geq 4 c a \Leftrightarrow 1-2 b+b^{2} \geq 4 c a \Leftrightarrow c-2 b c+b^{2} c \geq 4 a c^{2}$

Hay: $a+b+c-2(a b+b c+c a) \geq 3\left(a^{2} b+b^{2} c+c^{2} a\right)$

$\Leftrightarrow 1-2(a b+b c+c a) \geq 3\left(a^{2} b+b^{2} c+c^{2} a\right)$

Vậy: $F \geq 14[1-2(a b+b c+c a)]+\frac{3(a b+b c+c a)}{1-2(a b+b c+c a)}$

Đạt: $t=1-2(a b+b c+c a), t \geq \frac{1}{3}$

Áp dụng bất đẳng thức Cauchy ta có:

$F \geq 14 t+\frac{\frac{3}{2}(1-t)}{t}=14 t+\frac{3}{2 t}-\frac{3}{2}=\frac{1}{2} t+\frac{27}{2} t+\frac{3}{2 t}-\frac{3}{2} \geq \frac{1}{2} t+2 \sqrt{\frac{27}{2} t \cdot \frac{3}{2 t}}-\frac{3}{2}$

Vậy: $F \geq \frac{1}{2} \cdot \frac{1}{3}+9-\frac{3}{2}=\frac{23}{3}$

Đẳng thức xảy ra khi: $a=b=c=\frac{1}{3}$

Bài 4. Cho tứ giác $A B C D$ nội tiếp đường tròn $(O, R)$ có $A C$ vuông góc với $B D$ tại $H$. Trên cạnh $A B$ lấy điểm $M$ sao cho $A B=3 A M$. Trên cạnh $H C$ lấy trung điểm $N$. Chứng minh rằng $M H$ vuông góc với $D N$.

Lời giải.

  • Gọi $K, L$ lần lượt là trung điểm $B M$ và $H B, P$ là giao điểm của $H M$ và $A K$.

  • Ta có $K L$ là đường trung bình của tam giác $H M B$ nên $K L$ song song $H M$. Khi đó xét tam giác $A K L$ thì $P H$ là đường trung bình nên $P$ là trung điểm của $A K$.

  • Ta có từ $A B C D$ nội tiếp suy ra $H D \cdot H B=H A \cdot A C \Rightarrow H K \cdot H D=H A \cdot H N$, do đó $A D N K$ nội tiếp.

  • Suy ra $\angle N H Q=\angle A H P=\angle H A P=\angle H D N$, suy ra $\angle H Q N=90^{\circ}$.

Bài 5. Cho đường tròn tâm $O$ và đường tròn tâm $I$ cắt nhau tại hai điểm $A$ và $B(O$ và $I$ nằm khác phía đối với đường thẳng $A B), I B$ cắt $(O)$ tại $E, O B$ cắt $(I)$ tại $F$. Qua $B$ vẽ đường thẳng $M N$ song song với $E F(M$ thuộc $(O), N$ thuộc $(I))$.

a) Chứng minh rằng $O A I E$ nội tiếp.

b) Chứng minh rằng: $A E+A F=M N$.

Lời giải.

a) Chứng minh rằng tứ giác $A O E F$ nội tiếp

Do hai đường tròn $(\mathrm{O})$ và $(\mathrm{I})$ cắt nhau tại $A$ và $B$ nên ta có: $A$ đối xứng với $B$ qua $O I$. Vậy: $\angle O A I=\angle O B I$

Ta có tam giác $\triangle O B E$ cân tại $O$ nên $\angle O B E=\angle O E B$, do $\angle O B E+\angle O B I=180^{\circ}$ nên $\angle O E B+\angle O B I=180^{\circ}$. Từ đó ta có: $\angle O E B+\angle O A I=180^{\circ}$

Vậy tứ giác $O A I E$ là tứ giác nội tiếp. Chứng minh tương tự ta có: tứ giác $O A I F$ là tứ giác nội tiếp.

$\angle O E A=\angle O I A$ (tứ giác $O A I E$ là tứ giác nội tiếp)

$\angle O I A=\angle O F A$ (tứ giác $O A I F$ là tứ giác nội tiếp)

Vậy: $\angle O E A=\angle O F A$ nên tứ giác $O A F E$ là tứ giác nội tiếp

b) Chứng minh rằng: $M N=A E+A F$

Bài toán cần chứng minh tương đương với: $A F=B N$ và $A E=B M$.

Ta chỉ cần chứng minh $A F=B N$ vì $A E=B M$ là điều tương tự.

Để chứng minh $A F=B N$. Ta chỉ cần chứng minh số đo cung $\mathrm{AF}$ bằng số đo cung $\mathrm{BN}(A F, B N$ lần lượt là dây căng cung $\mathrm{AF}$, cung $\mathrm{BN}$ trong đường tròn (I)). Hay chỉ cần chứng minh: số đo cung $\mathrm{AB}$ bằng số đo cung FN. Từ đó ta chứng minh: $\angle O F A=\angle F B N$ là bài toán được giải quyết.

Do $E F | M N$ nên ta có: $\angle O F E=\angle F B N$

Mà $\angle O F E=\angle O A E=\angle O E A=\angle O F A$ (tứ giác $A O E F$ là tứ giác nội tiếp)

Từ đó ta có: $\angle O F A=\angle F B N$ (đpcm)

Bài 6. Trên mặt phẳng cho 2013 điểm tùy ý sao cho trong ba điểm bất kì thì tồn tại 2 điểm mà khoảng cách giữa hai điểm đó luôn bé hơn 1. Chứng minh rằng tồn tại một đường tròn có bán kính bằng 1 chứa ít nhất 1007 điểm (kế cả biên).

Lời giải. Gọi $A$ là một điểm bất kì trong 2013 điểm trên. Lấy $A$ làm tâm vẽ đường tròn có bán kính bằng 1 .

Nếu 2012 điểm còn lại thuộc đường tròn $(A)$ thì bài toán được chứng minh xong. Giả tồn tại một số điểm nằm ngoài đường tròn tâm $(A)$. Lấy điểm $(B)$ bất kì trong các điểm đó và vẽ đường tròn tâm $(B)$ có bán kính bằng 1 .

Giả sử tồn tại một điểm $C$ nằm ngoài hai đường tròn $(A)$ và $(B)$ thì $A B, A C$ đều lớn hơn 1. Điều này vô lí.

Từ đó ta có tất cả các điểm đã cho đều thuộc trong hai đường tròn $(A)$ và $(B)$.

Theo nguyên lí Dirichlet sẽ tồn tại một đường tròn chứa $\frac{2012}{2}+1=1007$ điểm (đpcm).

 

 

 

 

 

 

 

 

 

 

 

 

 

 

 

 

 

 

 

 

 

 

 

 

 

 

 

 

 

 

 

 

 

 

 

 

 

 

 

 

 

 

 

 

 

 

 

 

 

 

 

ĐỀ THI VÀO LỚP 10 CHUYÊN TOÁN TP.HCM 2013

Bài 1. (a) Giải phương trình: $x \sqrt{2 x-2}+5 x=9$.

(b) Cho $x, y, z$ đôi một khác nhau thỏa mãn: $\frac{1}{x}+\frac{1}{y}+\frac{1}{z}=0$. Tính giá trị biểu thực:

$P=\frac{y z}{x^{2}+2 y z}+\frac{z x}{y^{2}+2 z x}+\frac{x y}{z^{2}+2 x y}$

Bài 2. Cho phương trình $x^{2}-5 m x-4 m=0$.

(a) Định $m$ để phương trình có hai nghiệm phân biệt.

(b) Gọi $x_{1}, x_{2}$ là hai nghiệm của phương trình. Tìm $m$ để biểu thức sau đạt giá trị nhỏ nhất:

$\frac{m^{2}}{x_{1}^{2}+5 m x_{2}+12 m}+\frac{x_{2}^{2}+5 m x_{1}+12 m}{m^{2}}$

Bài 3. Cho tam giác $\triangle A B C$ có $B C$ là cạnh dài nhất. Trên $B C$ lấy hai điểm $D$ và $E$ sao cho $B D=B A, C E=C A$. Đường thẳng qua $D$ song song với $A B$ cắt $A C$ tại $M$. Đường thẳng qua $E$ song song với $A C$ cắt $A B$ tại $N$. Chứng minh rằng $A M=A N$.

Bài 4. Cho $x, y$ là hai số dương thỏa mãn: $x+y=1$. Chứng minh: $3(3 x-2)^{2}+\frac{8 x}{y} \geq$ $7 .$

Bài 5. Từ một điểm $A$ bên ngoài đường tròn $(O)$ vẽ các tiếp tuyến $A B, A C$ và cát tuyến $A E F$ (EF không đi qua $O, B$ và $C$ là các tiếp điểm). Gọi $D$ là điểm đôi xứng của $B$ qua $O . D E, D F$ lần lượt cắt $A O$ tại $M$ và $N$. Chứng minh rằng :

(a) Hai tam giác $\triangle C E F$ và $\triangle C M N$ đồng dạng.

(b) $O M=O N$.

Bài 6. Chữ số hàng đơn vị trong hệ thập phân của số $M=a^{2}+a b+b^{2}$ là $0\left(a ; b \in N^{*}\right)$.

(a) Chứng minh rằng $M$ chia hết cho 20 .

(b) Tìm chữ số hàng chục của $M$.

LỜI GIẢI

Bài 1.

a) Giải phương trình: $x \sqrt{2 x-2}+5 x=9$.

b) Cho $x, y, z$ đôi một khác nhau thỏa mãn: $\frac{1}{x}+\frac{1}{y}+\frac{1}{z}=0$. Tính giá trị biểu thực:

$P=\frac{y z}{x^{2}+2 y z}+\frac{z x}{y^{2}+2 z x}+\frac{x y}{z^{2}+2 x y}$

Lời giải.

a) Giải phương trình: $x \sqrt{2 x-2}+5 x=9$

ĐKXĐ: $x \geq 1$. Đặt $a=\sqrt{2 x-2}$ (ĐKXĐ: $a \geq 0$ )

Phương trình đã cho tương đương với:

$a x=9-5 x=9-\frac{5}{2}\left(a^{2}+2\right)=4-\frac{5}{2} a^{2}$

Ta có hệ phương trình sau:

$\left\{\begin{array} { l }{ 5 a ^ { 2 } + 2 a x = 8 } \\{ a ^ { 2 } – 2 x = – 2 }
\end{array} \Leftrightarrow \left\{\begin{array}{l}
x=\frac{a^{2}+2}{2} \\x=\frac{9}{a+5}
\end{array}\right.\right.$

$\Leftrightarrow \frac{9}{a+5}=\frac{a^{2}+2}{2} \Leftrightarrow a^{3}+5 a^{2}+2 a-8=0 \Leftrightarrow(a-1)(a+2)(a+4)=0$

Kết hợp với: ĐKXĐ: $a \geq 0$. Từ đó ta tính được: $a=1 \Leftrightarrow x=\frac{3}{2}$

b) Tính giá trị biểu thức: $P=\frac{y z}{x^{2}+2 y z}+\frac{z x}{y^{2}+2 z x}+\frac{x y}{z^{2}+2 x y}$

Từ điều kiện của đề bài ta có: $x y+y z+z x=0$

Thêm vào đó: $x^{2}+2 y z=x^{2}+y z-x y-x z=(x-y)(x-z)$

Từ đó ta có:

$P=\sum_{x, y, z} \frac{y z}{x^{2}+2 y z}=\sum_{x, y, z} \frac{y z}{(x-y)(x-z)}=-\frac{y z(y-z)+x z(z-x)+x y(x-y)}{(x-y)(y-z)(z-x)}$

Vậy: $P=1$

Bài 2. Cho phương trình $x^{2}-5 m x-4 m=0$.

a) Định $m$ để phương trình có hai nghiệm phân biệt.

b) Gọi $x_{1}, x_{2}$ là hai nghiệm của phương trình. Tìm $m$ để biểu thức sau đạt giá trị nhỏ nhất:

$\frac{m^{2}}{x_{1}^{2}+5 m x_{2}+12 m}+\frac{x_{2}^{2}+5 m x_{1}+12 m}{m^{2}}$

Lời giải.

a) Định $m$ để phương trình có hai nghiệm phân biệt

ĐKXĐ đề phương trình có hai nghiệm phân biệt là:

$\Delta=(-5 m)^{2}-4(-4 m)=25 m^{2}+16 m=m(25 m+16)>0$

$\Leftrightarrow\left\{\begin{array}{l}m>0 \\ m<\frac{-16}{25}\end{array}\right.$

b) Tìm $m$ để biếu thức sau đạt giá trị nhỏ nhất:

$P=\frac{m^{2}}{x_{1}^{2}+5 m x_{2}+12 m}+\frac{x_{2}^{2}+5 m x_{1}+12 m}{m^{2}}$

Do $x_{1}, x_{2}$ là hai nghiệm của phương trình nên ta có: $\left\{\begin{array}{l}x_{1}^{2}=5 m x_{1}+4 m \\ x_{2}^{2}=5 m x_{2}+4 m\end{array}\right.$

Do phương trình đã cho có hai nghiệm phân biệt nên: $25 m^{2}+16 m>0$. Từ đó áp dụng bất đẳng thức Cauchy, ta có:

$P=\frac{m^{2}}{x_{1}^{2}+5 m x_{2}+12 m}+\frac{x_{2}^{2}+5 m x_{1}+12 m}{m^{2}}$

$P=\frac{m^{2}}{25 m^{2}+16 m}+\frac{25 m^{2}+16 m}{m^{2}} \geq 2$

Đẳng thức xảy ra khi và chỉ khi: $m^{2}=25 m^{2}+16 m \Leftrightarrow m=\frac{-2}{3}$

Bài 3. Cho tam giác $\triangle A B C$ có $B C$ là cạnh dài nhất. Trên $B C$ lấy hai điểm $D$ và $E$ sao cho $B D=B A, C E=C A$. Đường thẳng qua $D$ song song với $A B$ cắt $A C$ tại $M$. Đường thẳng qua $E$ song song với $A C$ cắt $A B$ tại $N$. Chứng minh rằng $A M=A N$.

Lời giải.

Do $D M | A B$, áp dụng định lí Talet:

$\frac{A M}{A C}=\frac{B D}{B C} \Leftrightarrow A M=\frac{B D}{B C} \cdot A C=\frac{B A \cdot A C}{B C}$

Do $E N | A C$, áp dụng định lí Talet:

$\frac{A N}{A B}=\frac{C E}{B C} \Leftrightarrow A N=\frac{C E}{B C} \cdot A B=\frac{B A \cdot A C}{B C}$

Từ đó ta có $A M=A N$. Đây chính là điều phải chứng minh.

Bài 4. Cho $x, y$ là hai số dương thỏa mãn: $x+y=1$. Chứng minh: $3(3 x-2)^{2}+\frac{8 x}{y} \geq 7$.

Lời giải. Do $x+y=1$ nên ta có điều phải chứng minh trở thành:

$3(3 x-2)^{2}+\frac{8 x}{1-x} \geq 7$

Bằng khai triển và biến đổi tương đương ta có: $(5-3 x)(3 x-1)^{2} \geq 0$. Bất đẳng thức này hiển nhiên đúng do $x<1$

Bài 5.Từ một điểm $A$ bên ngoài đường tròn $(O)$ vẽ các tiếp tuyến $A B, A C$ và cát tuyến $A E F$ ( $E F$ không đi qua $O, B$ và $C$ là các tiếp điểm). Gọi $D$ là điểm đối xứng của $B$ qua $O$. $D E, D F$ lần lượt cắt $A O$ tại $M$ và $N$. Chứng minh rằng :

a) Hai tam giác $\triangle C E F$ và $\triangle C M N$ đồng dạng.

b) $O M=O N$.

Lời giải.
a) Chứng minh rằng $\triangle C E F \backsim \triangle C M N$
Ta có: $A N | C D$ (cùng vuông góc với $B C$ )
$\angle D F C=\angle D B C=\angle B A O=\angle C A O$
Từ đó ta có: tứ giác $C F N A$ nội tiếp
Vậy: $\angle C F E=\angle C N M$
Ta có: $A N | C D$ nên: $\angle O M E=\angle C D E$
Do tứ giác $C D F E$ nội tiếp nên: $\angle C D E=\angle C F E$
Vậy: $\angle O M E=\angle C F E$
Mà: $\angle A C E=\angle C F E$ (Tính chất tiếp tuyến)
Từ đó ta có: $\angle A C E=\angle O M E$. Vậy tứ giác $A M E C$ nội tiếp. Nên: $\angle E A M=$ $\angle E C M$

Mà: $\angle E A M=\angle F C N$ (Tứ giác $A N F C$ nội tiếp)

Vậy: $\angle E C M=\angle F C N$

Từ đó ta có: $\angle E C F=\angle M C N$

Do: $\angle C F E=\angle C N M$ và $\angle E C F=\angle M C N$ nên ta có: $\triangle C E F \sim \triangle C M N$

b) Chứng minh rằng: $O M=O N$

Từ giác $A M E C$ nội tiếp: $\angle D C M=\angle C A F$

Từ giác $C F N A$ nội tiếp: $\angle C A F=\angle C N D$

Vậy ta có: $\angle D C M=\angle C N D$ và do: $A N | C D$. Vậy $C D N M$ là hình thang cân nên: $C N=D M$ và $\angle C N M=\angle D M N$

Do $A O$ là đường trung trực của $B C$ nên ta có: $\angle C N M=\angle B N M$ và $N C=N B$

Từ đó ta có: $\angle D M N=\angle B N M$ và $D M=B N$

Hay: $D M | B N$ và $D M=B N$. Từ đó $B M D N$ là hình bình hành. Mà $O$ là trung điểm của $B D$ nên $O$ cũng là trung điểm của $M N$ hay: $O M=O N$ (đpcm)

Bài 6. Chữ số hàng đơn vị trong hệ thập phân của số $M=a^{2}+a b+b^{2}$ là $0\left(a ; b \in N^{*}\right)$.

a) Chứng minh rằng $M$ chia hết cho 20 .

b) Tìm chữ số hàng chục của $M$.

Lới giải.

a) Chứng minh rằng: $M \vdots 20$

Do chữ số hàng đơn vị của $M$ là 0 nên ta có: $M \vdots 5$ và $M \vdots 2$

Giả sử cả $a$ và $b$ đều không chia hết cho 2 . Từ đó ta có:

$\left\{\begin{array} { l }{ a \equiv 1 } \\ { b \equiv 1 }\end{array} \Rightarrow \left\{\begin{array}{l}a^{2} \equiv 1 \\ b^{2} \equiv 1 \\ a b \equiv 1\end{array} \Rightarrow a^{2}+a b+b^{2} \equiv 1 \Rightarrow M \equiv 1(\bmod 2)\right.\right.$

Điều này vô lí: từ đó ta có trong hai số $a$ và $b$ phải có một số chia hết cho 2 .

Giả sử $a \vdots$ 2. Do $M \vdots 2$ nên $b^{2} \vdots 2$. Từ đó ta có: $b \vdots 2$

Vi $a \vdots 2$ và $b \vdots 2$ nên $M \vdots 4$

Do $M \vdots 4$ và $M \vdots 5$ nên ta có: $M \vdots 20$ (đpcm)

b) Nhận xét: Một số chính phương khi chia cho 5 dư 0,1 hoặc 4 .

Ta có $5 \mid a^{2}+a b+b^{2}$, suy ra $5 \mid 4 a^{2}+4 a b+4 b^{2}$ hay $5 \mid(2 a+b)^{2}+3 b^{2}$.

Từ nhận xét trên suy ra $5|b, 5| 2 a+b \Rightarrow 5 \mid a$. Do đó $a^{2}+a b+b^{2}$ chia hết cho $25 .$

Kết hợp với câu a ta có $M$ chia hết cho 100 nên chữ số hàng chục là số 0 .

 

 

 

 

 

 

 

 

 

 

 

 

 

 

ĐỀ THI VÀO LỚP 10 CHUYÊN TOÁN TP.HCM – NĂM 2014

Bài 1. (a) Giải phương trình: $x \sqrt{2 x-3}=3 x-4$

(b) Cho 3 số thực $x, y, z$ thỏa mãn điều kiện: $x+y+z=0 ; x y z \neq 0$. Tính giá trị biểu thức:

$P=\frac{x^{2}}{y^{2}+z^{2}-x^{2}}+\frac{y^{2}}{z^{2}+x^{2}-y^{2}}+\frac{z^{2}}{x^{2}+y^{2}-z^{2}}$

Bài 2. Giải hệ phương trình: $\left\{\begin{array}{l}x+y+\frac{1}{y}=\frac{9}{x} \\ x+y-\frac{4}{x}=\frac{4 y}{x^{2}}\end{array}\right.$

Bài 3. Cho tam giác đều $A B C$ và $M$ là một điểm bất kì trên cạnh $B C$. Gọi $D, E$ lần lượt là hình chiếu vuông góc của $M$ trên $A B$ và $A C$. Xác định vị trí của $M$ để tam giác $M D E$ có chu vi nhỏ nhất.

Bài 4. (a) Cho $x, y$ là 2 số thực khác 0 . Chứng minh rằng: $\frac{x^{2}}{y^{2}}+\frac{y^{2}}{x^{2}} \geq \frac{x}{y}+\frac{y}{x}$

(b) Cho $a, b$ là hai số dương. Tìm giá trị nhỏ nhất của biểu thức: $P=\frac{a^{2}+3 a b+b^{2}}{\sqrt{a b}(a+b)}$

Bài 5. Từ một điểm $M$ nằm ngoài đường tròn $(\mathrm{O})$, kẻ các tiếp tuyến $M A, M B$ với $(\mathrm{O})$ $(A, B$ là các tiếp điểm $)$. Gọi $H$ là giao điểm của $A B$ với $O M, I$ là trung điểm của $M H$. Đường thẳng $A I$ cắt $(\mathrm{O})$ tại điểm $K(K$ khác $A)$.

(a) Chứng minh $H K$ vuông góc với $A I$.

(b) Tính số đo góc $\angle M K B$.

Bài 6. Tìm cặp số nguyên $(x, y)$ thỏa mãn phương trình:

$2015\left(x^{2}+y^{2}\right)-2014(2 x y+1)=25$

LỜI GIẢI

 

Bài 1.

a) Giải phương trình: $x \sqrt{2 x-3}=3 x-4$

b) Cho 3 số thực $x, y, z$ thỏa mãn điều kiện: $x+y+z=0 ; x y z \neq 0$. Tính giá trị biểu thức:

$P=\frac{x^{2}}{y^{2}+z^{2}-x^{2}}+\frac{y^{2}}{z^{2}+x^{2}-y^{2}}+\frac{z^{2}}{x^{2}+y^{2}-z^{2}}$

Lời giải.

a) Giải phương trình: $x \sqrt{2 x-3}=3 x-4 Đ \mathrm{~K} Đ: x \geq \frac{3}{2}$

Phương trình đã cho tương đương với:

$x^{2}(2 x-3)=9 x^{2}-24 x+16 \Leftrightarrow 2 x^{3}-12 x^{2}+24 x-16=0 $

$\Leftrightarrow x^{3}-6 x^{2}+12 x-8=0 \Leftrightarrow(x-2)^{3}=0 \Leftrightarrow x=2$

Ta thấy $x=2$ thỏa yêu cầu bài toán, vậy $x=2$ là nghiệm duy nhất của phương trình.

b) Cho 3 số thực $x, y, z$ thỏa mãn điều kiện: $x+y+z=0 ; x y z \neq 0$. Tính giá trị biểu thức:

$P=\frac{x^{2}}{y^{2}+z^{2}-x^{2}}+\frac{y^{2}}{z^{2}+x^{2}-y^{2}}+\frac{z^{2}}{x^{2}+y^{2}-z^{2}}$

Ta có:

$y+z=-x \Leftrightarrow y^{2}+2 y z+z^{2}=x^{2} \Leftrightarrow y^{2}+z^{2}-x^{2}=-2 y z $

$x+z=-y \Leftrightarrow x^{2}+2 x z+z^{2}=y^{2} \Leftrightarrow x^{2}+z^{2}-y^{2}=-2 x z $

$y+x=-z \Leftrightarrow y^{2}+2 y x+x^{2}=z^{2} \Leftrightarrow y^{2}+x^{2}-z^{2}=-2 y x$

Từ đó ta tính được $P$ :

$P=\frac{x^{2}}{-2 y z}+\frac{y^{2}}{-2 x z}+\frac{z^{2}}{-2 y x}=\frac{x^{3}+y^{3}+z^{3}}{-2 x y z}$

Chú ý:

$x^{3}+y^{3}+z^{3}-3 x y z=0 \Rightarrow x^{3}+y^{3}+z^{3}=3 x y z$

Vậy: $P=\frac{x^{3}+y^{3}+z^{3}}{-2 x y z}=\frac{3 x y z}{-2 x y z}=\frac{-3}{2}$

Bài 2. Giải hệ phương trình: $\left\{\begin{array}{l}x+y+\frac{1}{y}=\frac{9}{x} \\ x+y-\frac{4}{x}=\frac{4 y}{x^{2}}\end{array}\right.$

Lời giải. ĐKXĐ: $x, y \neq 0$

Lấy phương trình (1) trừ phương trình (2) ta thu được:

$\frac{1}{y}+\frac{4}{x}=\frac{9}{x}-\frac{4 y}{x^{2}}  \Leftrightarrow \frac{1}{y}=\frac{5}{x}-\frac{4 y}{x^{2}} \Leftrightarrow x^{2}=5 x y-4 y^{2} \Leftrightarrow x^{2}-5 x y+4 y^{2}=0 $

$\Leftrightarrow(x-4 y)(x-y)=0 \Leftrightarrow\left[\begin{array}{l}x=4 y \\ x=y\end{array}\right.$

Trường hợp 1: $x=4 y$. Thay vào phương trình (1) ta có:

$5 y+\frac{1}{y}=\frac{9}{4 y} \Leftrightarrow 5 y=\frac{5}{4 y} \Leftrightarrow\left[\begin{array} { l }{ y = \frac { 1 } { 2 } } \\ { y = \frac { – 1 } { 2 } }\end{array} \Leftrightarrow \left[\begin{array}{l}x=2, y=\frac{1}{2} \\ x=-2, y=\frac{-1}{2}\end{array}\right.\right.$

Trường hợp $2: x=y$. Thay vào phương trình (1) ta có:

$2 y+\frac{1}{y}=\frac{9}{y} \Leftrightarrow 2 y=\frac{8}{y} \Leftrightarrow\left[\begin{array}{l}y=2 \\ y=-2\end{array} \Leftrightarrow\left[\begin{array}{l}x=2, y=2 \\ x=-2, y=-2\end{array}\right.\right.$

Vậy tập nghiệm của phương trình là: $(x, y)=(2,2),(-2,-2),\left(2, \frac{1}{2}\right),\left(-2, \frac{-1}{2}\right)$

Bài 3. Cho tam giác đều $A B C$ và $M$ là một điểm bất kì trên cạnh $B C$. Gọi $D, E$ lần lượt là hình chiếu vuông góc của $M$ trên $A B$ và $A C$. Xác định vị trí của $M$ để tam giác MDE có chu vi nhỏ nhất.

Lời giải.

  • Gọi độ dài cạnh tam giác đều là $a$.

Ta có $M D \cdot A B+M E \cdot A C=2 S_{A M D}+2 S_{A M C}=2 S_{A B C}$. Hay $(M D+M E)=A H \cdot a$, suy ra $M D+M E=A H$ không đổi.

  • Ta có $D, E$ thuộc đường tròn đường kính $A M$. Vẽ đường kính $D F$, ta có $\angle D F E=$ $\angle D A E=60^{\circ}$.

Suy ra $D E=D F \sin D F E=A M \sin 60^{\circ}$.

$D E$ nhỏ nhất khi và chỉ khi $A M$ nhỏ nhất, khi và chỉ khi $M$ trùng với $H$ trung điểm $B C$.

  • Vậy chu vi tam giác $M D E$ nhỏ nhất khi và chỉ khi $M$ là trung điểm $B C$.

Bài 4.

a) Cho $x, y$ là 2 số thực khác 0 . Chứng minh rằng: $\frac{x^{2}}{y^{2}}+\frac{y^{2}}{x^{2}} \geq \frac{x}{y}+\frac{y}{x}$

b) Cho $a, b$ là hai số dương. Tìm giá trị nhỏ nhất của biểu thức:

$P=\frac{a^{2}+3 a b+b^{2}}{\sqrt{a b}(a+b)}$

Lời giải.

a) Bằng biến đổi tương đương ta có:

$\frac{x^{2}}{y^{2}}+\frac{y^{2}}{x^{2}}-\left(\frac{x}{y}+\frac{y}{x}\right) \geq 0 \Leftrightarrow \frac{(x-y)^{2}\left(\left(x+\frac{1}{2}\right)^{2}+\frac{3}{4} y^{2}\right)}{x^{2} y^{2}} \geq 0$

Bất đẳng thức cuối luôn đúng. Dấu bằng trong bất đẳng thức xảy ra khi $x=y$.

b) Cách 1: Với $a, b$ là hai số dương. Ta có:

$P=\frac{a^{2}+3 a b+b^{2}}{\sqrt{a b}(a+b)}=\frac{(a+b)^{2}+a b}{\sqrt{a b}(a+b)}=\frac{\frac{1}{4}(a+b)^{2}+a b+\frac{3}{4}(a+b)^{2}}{\sqrt{a b}(a+b)} $

$P=\frac{\frac{1}{4}(a+b)^{2}+a b}{\sqrt{a b}(a+b)}+\frac{\frac{3}{4}(a+b)}{\sqrt{a b}}$

Áp dụng bất đẳng thức Cauchy:

$P=\frac{\frac{1}{4}(a+b)^{2}+a b}{\sqrt{a b}(a+b)}+\frac{\frac{3}{4}(a+b)}{\sqrt{a b}} \geq \frac{2 \sqrt{\frac{1}{4} a b(a+b)^{2}}}{\sqrt{a b}(a+b)}+\frac{\frac{3}{4} \cdot 2 \sqrt{a b}}{\sqrt{a b}}=1+\frac{3}{2}=\frac{5}{2}$

Dấu bằng trong bất đẳng thức xảy ra khi $a=b$

Cách 2: Ta có:

$P=\frac{a^{2}+3 a b+b^{2}}{\sqrt{a b}(a+b)}=\frac{(a+b)^{2}+a b}{\sqrt{a b}(a+b)}=\frac{a+b}{\sqrt{a b}}+\frac{\sqrt{a b}}{a+b}=\frac{3}{4} \cdot \frac{a+b}{\sqrt{a b}}+\frac{1}{4} \cdot \frac{a+b}{\sqrt{a b}}+\frac{\sqrt{a b}}{a+b}$

Áp dụng bất đẳng thức Cauchy:

$P \geq \frac{3}{4} \cdot 2+2 \sqrt{\frac{1}{4} \cdot \frac{a+b}{\sqrt{a b}} \cdot \frac{\sqrt{a b}}{a+b}}=\frac{3}{2}+1=\frac{5}{2}$

Dấu bằng trong bất đẳng thức xảy ra khi $a=b$

Bài 5.Từ một điểm $M$ nằm ngoài đường tròn $(\mathrm{O})$, kẻ các tiếp tuyến $M A, M B$ với $(\mathrm{O})$ $(A, B$ là các tiếp điểm $)$. Gọi $H$ là giao điểm của $A B$ với $O M, I$ là trung điểm của $M H$. Đường thẳng $A I$ cắt $(\mathrm{O})$ tại điểm $K(K$ khác $A)$.

a) Chứng minh $H K$ vuông góc với $A I$.

b) Tính số đo góc $\angle M K B$.

Lời giải.

a) Vẽ đường kính $A C, C H$ cắt $A I$ tại $K^{\prime}$.

Dễ thấy hai tam giác $A B C$ và $M H A$ đồng dạng, từ đó suy ra $A C H$ và $M A I$ đồng dạng.

Suy ra $\angle A C H=\angle M A I$, mà $\angle M A I+\angle I A C=90^{\circ}$, suy ra $\angle A C H+\angle I A C=$ $90^{\circ}$.

Do đó $\angle A K^{\prime} C=90^{\circ}$, suy ra $K^{\prime}$ thuộc $(O)$, từ đó $K^{\prime} \equiv K$. Ta có điều cần chứng minh.

b) Ta có $I K \cdot I A=I H^{2}=I M^{2}$.

Suy ra $\triangle I K M \backsim \triangle I M A$, do đó $\angle I M K=\angle I A M=\angle K B H$.

Từ đó tứ giác $B H K M$ nội tiếp, suy ra $\angle B K M=\angle B H M=90^{\circ}$.

Bài 6. Tìm cặp số nguyên $(x, y)$ thỏa mãn phương trình:

$2015\left(x^{2}+y^{2}\right)-2014(2 x y+1)=25$

Lời giải.

Ta có: $2015\left(x^{2}+y^{2}\right)-2014(2 x y+1)=25$

$\Leftrightarrow 2014(x-y)^{2}+x^{2}+y^{2}=2039$

Vậy: $2014(x-y)^{2} \leq 2039 \Leftrightarrow|x-y| \leq 1$

  • Trường hợp 1: $x-y=0$. Ta có: $x^{2}+y^{2}=2039$

Phương trình này không có nghiệm nguyên vì 2039 không chia hết cho $2 .$

  • Trường hợp 2: $x-y=1$. Ta có: $y^{2}+y-12=0$

Phương trình này có nghiệm $y=3$ hay $y=-4$

Từ đó ta có hai cặp nghiệm của phương trình là: $(x, y)={(4 ; 3),(-3 ;-4)}$

  • Trường hợp 3: $x-y=-1$. Ta có: $y^{2}-y-12=0$

Phương trình này có nghiệm $y=-3$ hay $y=4$

Từ đó ta có hai cặp nghiệm của phương trình là: $(x, y)={(-4 ;-3),(3 ; 4)}$

Vậy tập nghiệm của phương trình là: $(x, y)={(4 ; 3),(-3 ;-4),(3 ; 4),(-4 ;-3)}$

 

 

 

 

 

 

 

 

 

 

 

 

 

 

 

 

 

 

 

 

 

 

 

 

 

 

 

 

 

ĐỀ THI VÀO LỚP 10 CHUYÊN TOÁN TP.HCM – NĂM 2015

Bài 1. Cho hai số thực $a, b$ thỏa điều kiện $a b=1, a+b \neq 0$. Tính giá trị của biểu thức:

$P=\frac{1}{(a+b)^{3}}\left(\frac{1}{a^{3}}+\frac{1}{b^{3}}\right)+\frac{3}{(a+b)^{4}}\left(\frac{1}{a^{2}+b^{2}}\right)+\frac{6}{(a+b)^{5}}\left(\frac{1}{a}+\frac{1}{b}\right)$

Bài 2. (a) Giải phương trình: $2 x^{2}+x+3=3 x \sqrt{x+3}$

(b) Chứng minh rằng: $a b c\left(a^{3}-b^{3}\right)\left(b^{3}-c^{3}\right)\left(c^{3}-a^{3}\right)$ chia hết cho 7 với mọi số nguyên $a, b, c$

Bài 3. Cho hình bình hành $A B C D$. Đường thẳng qua $C$ vuông góc với $C D$ cắt đường thẳng qua $A$ vuông góc với $B D$ tại $F$. Đường thẳng qua $B$ vuông góc với $A B$ cắt đường trung trực của $A C$ tại $E$. Hai đường thẳng $B C$ và $E F$ cắt nhau tại $K$. Tính tỉ số: $\frac{K E}{K F}$.

Bài 4. Cho hai số dương $a, b$ thỏa mãn điều kiện: $a+b \leq 1$. Chứng minh rằng: $a^{2}-$ $\frac{3}{4 a}-\frac{a}{b} \leq-\frac{9}{4}$

Bài 5. Cho tam giác $\triangle A B C$ có ba góc nhọn nội tiếp đường tròn $(\mathrm{O})$. Gọi $M$ là trung điểm của cạnh $B C$ và $N$ là điểm đối xứng của $M$ qua $O$. Đường thẳng qua $A$ vuông góc với $A N$ cắt đường thẳng qua $B$ vuông góc với $B C$ tại $D$. Kẻ đường kính $A E$.

(a) Chứng minh rằng: $B A \cdot B C=2 B D \cdot B E$

(b) $C D$ đi qua trung điểm của đường cao $A H$ của tam giác $\triangle A B C$.

Bài 6. Mười vận động viên tham gia cuộc thi đấu quần vợt. Cứ hai người trong họ chơi với nhau đúng một trận. Người thứ nhất thắng $x_{1}$ trận và thua $y_{1}$ trận, người thứ hai thắng $x_{2}$ và thua $y_{2}$ trận,… người thứ mười thắng $x_{10}$ trận và thua $y_{10}$. Biết rằng trong một trận đấu quần vợt không có kết quả hòa. Chứng minh rằng:

$x_{1}^{2}+x_{2}^{2}+\ldots+x_{10}^{2}=y_{1}^{2}+y_{2}^{2}+\ldots+y_{10}^{2}$

 

LỜI GIẢI

 

Bài 1. Cho hai số thực $a, b$ thỏa điều kiện $a b=1, a+b \neq 0$. Tính giá trị của biểu thức:

$P=\frac{1}{(a+b)^{3}}\left(\frac{1}{a^{3}}+\frac{1}{b^{3}}\right)+\frac{3}{(a+b)^{4}}\left(\frac{1}{a^{2}+b^{2}}\right)+\frac{6}{(a+b)^{5}}\left(\frac{1}{a}+\frac{1}{b}\right)$

Lời giải. Ta có: $a b=1$ và $a+b \neq 0$

$P=\frac{1}{(a+b)^{3}}\left(\frac{1}{a^{3}}+\frac{1}{b^{3}}\right)+\frac{3}{(a+b)^{4}}\left(\frac{1}{a^{2}}+\frac{1}{b^{2}}\right)+\frac{6}{(a+b)^{5}}\left(\frac{1}{a}+\frac{1}{b}\right)$ $=\frac{1}{(a+b)^{3}}\left(\frac{a^{3} b^{3}}{a^{3}}+\frac{a^{3} b^{3}}{b^{3}}\right)+\frac{3}{(a+b)^{4}}\left(\frac{a^{2} b^{2}}{a^{2}}+\frac{a^{2} b^{2}}{b^{2}}\right)+\frac{6}{(a+b)^{5}}\left(\frac{a b}{a}+\frac{a b}{b}\right)$ $=\frac{1}{(a+b)^{3}}\left(a^{3}+b^{3}\right)+\frac{3}{(a+b)^{4}}\left(a^{2}+b^{2}\right)+\frac{6}{(a+b)^{5}}(a+b)$ $=\frac{a^{2}-a b+b^{2}}{(a+b)^{2}}+\frac{3(a+b)^{2}-6}{(a+b)^{4}}+\frac{6}{(a+b)^{4}}$ $=\frac{a^{2}-a b+b^{2}}{(a+b)^{2}}+\frac{3}{(a+b)^{2}}=\frac{(a+b)^{2}}{(a+b)^{2}}$

Vậy P=1

Bài 2.

a) Giải phương trình: $2 x^{2}+x+3=3 x \sqrt{x+3}$

b) Chứng minh rằng: $a b c\left(a^{3}-b^{3}\right)\left(b^{3}-c^{3}\right)\left(c^{3}-a^{3}\right)$ chia hết cho 7 với mọi số nguyên $a, b, c$

Lời giải.

a) Điều kiện xác định: $x \geq-3$

$Ta có:  2 x^{2}+x+3=3 x \sqrt{x+3} $

$\Leftrightarrow 2 x^{2}-2 x \sqrt{x+3}-x \sqrt{x+3}+x+3=0 $

$\Leftrightarrow(x-\sqrt{x+3})(2 x-\sqrt{x+3})=0 $

$\Leftrightarrow\left[\begin{array} { l }{ x = \sqrt { x + 3 } ( x \geq 0 ) } \\ { 2 x = \sqrt { x + 3 } ( x \geq 0 ) }\end{array} \Leftrightarrow \left[\begin{array}{l}x^{2}-x-3=0(x \geq 0) \\ 4 x^{2}-x-3=0(x \geq 0)\end{array}\right.\right. $

$\Leftrightarrow\left[\begin{array}{l}x=\frac{1+\sqrt{13}}{2} \\ x=1\end{array}\right.$

b) Ta áp dụng bổ đề sau: Lập phương một số nguyên bất kì khi chia cho 7 đều chỉ có số dư là: $0,1,-1$.

Chứng minh tính chất này ta chỉ cần lập bảng số dư.

Nếu một trong ba số $a, b, c$ chia hết cho 7 , ta có điều cần chứng minh.

Nếu $a, b, c$ không có số nào chia hết cho 7 thì $a^{3}, b^{3}, c^{3}$ chia 7 dư $1,-1$, do đó theo nguyên lý Dirichlet thì có 2 số có hiệu chia hết cho 3, do đó ít nhất một trong các số $a^{3}-b^{3}, b^{3}-c^{3}, c^{3}-a^{3}$ chia hết cho 7 . Từ đó ta có điều cần chứng minh.

Bài 3. Cho hình bình hành $A B C D$. Đường thẳng qua $C$ vuông góc với $C D$ cắt đường thẳng qua $A$ vuông góc với $B D$ tại $F$. Đường thẳng qua $B$ vuông góc với $A B$ cắt đường trung trực của $A C$ tại $E$. Hai đường thẳng $B C$ và $E F$ cắt nhau tại $K$. Tính tỉ số: $\frac{K E}{K F}$.

Lời giải.

Gọi $M$ là giao điểm của $A F$ và $B D$ và $J$ là giao điểm của $A B$ và $O E$.

Ta có các tứ giác $A B E O, C D M F$ nội tiếp. Khi đó $\angle J E A=\angle J B O=180^{\circ}-\angle A B O=$ $180^{\circ}-\angle B D C=\angle A F C$

Và $\angle E J A=90^{\circ}-\angle J A O=\angle A C F$.

Khi đó $\triangle A J E \backsim \triangle A F C \Rightarrow \frac{A J}{A C}=\frac{J E}{C F} \Rightarrow \frac{A C}{C F}=\frac{A J}{J E}$. (1)

Mặt khác $\triangle A J O \backsim \triangle E J B \Rightarrow \frac{A J}{J E}=\frac{A O}{B E}$. (2)

Từ (1) và (2) ta có $\frac{A C}{C F}=\frac{A O}{B E} \Rightarrow \frac{B E}{C F}=\frac{A O}{A C}=\frac{1}{2}$.

Mặt khác $\frac{K E}{K F}=\frac{B E}{C F}=\frac{1}{2}$.

Bài 4. Cho hai số dương $a, b$ thỏa mãn điều kiện: $a+b \leq 1$. Chứng minh rằng: $a^{2}-$ $\frac{3}{4 a}-\frac{a}{b} \leq-\frac{9}{4}$

Lời giải. Do $a>0, b>0$ và $a+b \leq 1$. Ta chứng minh: $a^{2}-\frac{3}{4} a-\frac{a}{1-a} \leq-\frac{9}{4}$

$\Leftrightarrow 4 a^{4}-4 a^{3}+13 a^{2}-12 a+3 \geq 0 $

$\Leftrightarrow(2 a-1)^{2}\left(a^{2}+3\right) \geq 0 ${ (luôn đúng)

Dấu bằng trong bất đẳng thức này xảy ra khi: $a=b=\frac{1}{2}$

Bài 5. Cho tam giác $\triangle A B C$ có ba góc nhọn nội tiếp đường tròn $(\mathrm{O})$. Gọi $M$ là trung điểm của cạnh $B C$ và $N$ là điểm đối xứng của $M$ qua $O$. Đường thẳng qua $A$ vuông góc với $A N$ cắt đường thẳng qua $B$ vuông góc với $B C$ tại $D$. Kẻ đường kính $A E$.

a) Chứng minh rằng: $B A \cdot B C=2 B D \cdot B E$

b) $C D$ đi qua trung điểm của đường cao $A H$ của tam giác $\triangle A B C$.

Lời giải.

a) Chứng minh rằng: $B A \cdot B C=2 B D \cdot B E$

Điều này tương đương với: $B A . B M=B D . B E$

Xét hai tam giác $\triangle B M E$ và tam giác $\triangle B D A$ ta có:

$\angle D B A=\angle M B E$ (cùng phụ với góc $\angle A B C$ ) (1)

$\angle D A B+\angle B A E+\angle O A N=90^{\circ}$

Do $\triangle A O N=\triangle E O M$ nên $\angle O A N=\angle O E M$

Từ đó ta có: $\angle D A B+\angle B A E+\angle O E M=90^{\circ}$

Do $A E$ là đường kính của đường tròn $(O)$. Nên $\triangle A B E$ vuông tại $B$.

Từ đó ta có: $\angle B A E+\angle O E M+\angle B E M=90^{\circ}$

Vậy: $\angle D A B=\angle B E M$ (2)

Từ (1) và (2) ta có $\triangle B M E \backsim \triangle B D A$

Vậy: $\frac{B M}{B D}=\frac{B E}{B A}=\frac{M E}{D A}$ (đpcm)

b) Chứng minh rằng: $C D$ đi qua trung điểm $I$ của $A H$ Gọi $F$ là giao điểm của $B D$ và $C A$

Từ đó điều phải chứng minh tương đương với chứng minh rằng $D$ là trung điểm của $B F$. Mà $M$ là trung điểm của $B C$ như vậy ta chỉ cần chứng minh được: $M D | A C$

Xét hai tam giác vuông $\triangle B D M$ và tam giác $\triangle B A E$ ta có: $\frac{B D}{B M}=\frac{B A}{B E}$ Vậy: $\triangle B D M \backsim \triangle B A E$

Từ đó ta có: $\angle B M D=\angle B E A$

Mà: $\angle B E A=\angle B C A$ (cùng chắn cung $A B$ của đường tròn $(O)$ )

Vậy: $\angle B M D=\angle B C A$

Từ đó ta có: $M D | A C$. Đây cũng chính là điều phải chứng minh.

Bài 6. Mười vận động viên tham gia cuộc thi đấu quần vợt. Cứ hai người trong họ chơi với nhau đúng một trận. Người thứ nhất thắng $x_{1}$ trận và thua $y_{1}$ trận, người thứ hai thắng $x_{2}$ và thua $y_{2}$ trận,… người thứ mười thắng $x_{10}$ trận và thua $y_{10}$. Biết rằng trong một trận đấu quần vợt không có kết quả hòa. Chứng minh rằng:

$x_{1}^{2}+x_{2}^{2}+\ldots+x_{10}^{2}=y_{1}^{2}+y_{2}^{2}+\ldots+y_{10}^{2}$

Lời giải. Do trong một trận đấu chỉ có thắng hoặc thua nên tổng số trận thằng phải bằng với tổng số trận thua. Từ đó ta có:

$x_{1}+\ldots \ldots . .+x_{10}=y_{1}+\ldots \ldots . .+y_{10}$

Ta có tổng cộng là 45 trận $\left(\frac{10.9}{2}\right)$ nên tổng sổ trận thắng bằng tổng số trận thua bằng 45 trận.

$x_{1}+\ldots \ldots . .+x_{10}=y_{1}+\ldots \ldots . .+y_{10}=45$

Mỗi người sẽ thi đấu với 9 người còn lại nên:

$x_{i}+y_{i}=9 \Leftrightarrow x_{i}=9-y_{i} \Leftrightarrow x_{i}^{2}=81-18 y_{i}+y_{i}^{2}$

Từ đó ta có:

$\sum_{i=1}^{10} x_{i}^{2}=810-18 \sum_{i=1}^{10} y_{i}+\sum_{i=1}^{10} y_{i}^{2}=810-18.45+\sum_{i=1}^{10} y_{i}^{2}=\sum_{i=1}^{10} y_{i}^{2}$

Đây chính là điều phải chứng minh

Bài toán này cũng có thể tổng quát lên cho trường hợp $n$ người, phần này dành cho các em tự chứng minh.

 

 

 

 

 

 

 

 

 

 

 

 

 

 

 

 

 

 

 

 

 

ĐỀ THI VÀO LỚP 10 CHUYÊN TOÁN TP.HCM NĂM 2020

Bài 1. Cho ba số dương $a, b, c$ thỏa mãn điều kiện $\frac{a}{b+c}+\frac{b}{c+a}+\frac{c}{a+b}=2020$ Tính giá trị của biểu thức $P=\left(\frac{a^{2}}{b+c}+\frac{b^{2}}{c+a}+\frac{c^{2}}{a+b}\right):(a+b+c)$

Bài 2. (a) Giải phương trình: $\sqrt{2 x^{2}+x+9}+\sqrt{2 x^{2}-x+1}=x+4$

(b) Giải hệ phương trình: $\left\{\begin{array}{l}y^{2}-2 x y=8 x^{2}-6 x+1 \\ y^{2}=x^{3}+8 x^{2}-x+1\end{array}\right.$

Bài 3. Cho tam giác nhọn $A B C(A B<B C<C A)$ nội tiếp đường tròn $(O)$. Từ $A$ kẻ đường thẳng song song với $B C$ cắt $(O)$ tại $A_{1}$. Từ $B$ kẻ đường thẳng song song với $A C$ cắt $(O)$ tại $B_{1}$. Từ $C$ kẻ đường thẳng song song với $A B$ cắt $(O)$ tại $C_{1}$. Chứng minh rằng các đường thẳng qua $A_{1}, B_{1}, C_{1}$ lần lượt vuông góc với $B C$, $C A, A B$ đồng quy.

Bài 4. (a) Cho 2 số thực $a, b$. Chứng minh rằng: $\frac{a^{2}+b^{2}}{2} \geq a b+\frac{(a-b)^{2}}{a^{2}+b^{2}+2}$

(b) Cho hai số dương $a, b$ thỏa mãn điều kiện $a+b \leq 3$

Tìm giá trị nhỏ nhất của biểu thức: $Q=b-a+\frac{20}{a}+\frac{7}{b}$.

Bài 5. Đường tròn $(I)$ nội tiếp tam giác $A B C$ tiếp xúc với các cạnh $A B, B C, C A$ lần lượt tại $D, E, F$. Kẻ đường kính $E J$ của đường tròn $(I)$. Gọi $d$ là đường thẳng qua $A$ song song với $B C$. Đường thẳng $J D$ cắt $d, B C$ lần lượt tại $L, H$.

(a) Chứng minh: $E, F, L$ thẳng hàng.

(b) $J A, J F$ cắt $B C$ lần lượt tại $M, K$. Chứng minh: $M H=M K$.

Bài 6. Tìm tất cả các số nguyên dương $x, y$ thỏa mãn phương trình: $3^{x}-y^{3}=1$

 

LỜI GIẢI

 

Bài 1. Cho ba số dương $a, b, c$ thỏa mãn điều kiện $\frac{a}{b+c}+\frac{b}{c+a}+\frac{c}{a+b}=2020$ Tính giá trị của biểu thức $P=\left(\frac{a^{2}}{b+c}+\frac{b^{2}}{c+a}+\frac{c^{2}}{a+b}\right):(a+b+c)$

Lời giải.

Ta có: $\left(\frac{a}{b+c}+\frac{b}{c+a}+\frac{c}{a+b}\right)(a+b+c)=2020(a+b+c)$

$\Leftrightarrow \frac{a^{2}}{b+c}+a+b+\frac{b^{2}}{c+a}+\frac{c^{2}}{a+b}+c=2020(a+b+c)$ $\Leftrightarrow \frac{a^{2}}{b+c}+\frac{b^{2}}{c+a}+\frac{c^{2}}{a+b}=2019(a+b+c)$ $\Leftrightarrow\left(\frac{a^{2}}{b+c}+\frac{b^{2}}{c+a}+\frac{c^{2}}{a+b}\right):(a+b+c)=2019$ Vạy $P=2019 .$

Bài 2.

a) Giải phương trình: $\sqrt{2 x^{2}+x+9}+\sqrt{2 x^{2}-x+1}=x+4$

b) Giải hệ phương trình: $\left\{\begin{array}{l}y^{2}-2 x y=8 x^{2}-6 x+1 \\ y^{2}=x^{3}+8 x^{2}-x+1\end{array}\right.$

Lời giải.

a) $\sqrt{2 x^{2}+x+9}+\sqrt{2 x^{2}-x+1}=x+4$ (1)

Đặt $\sqrt{2 x^{2}+x+9}=a(a>0)\left(\right.$ do $\left.2 x^{2}+x+9>0\right)$

và $\sqrt{2 x^{2}-x+1}=b(b>0)\left(\right.$ do $\left.2 x^{2}-x+1>0\right)$

Khi đó ta có: $a^{2}-b^{2}=2 x+8$

Thay vào phương trình ta có:

$a+b=\frac{a^{2}-b^{2}}{2} \Leftrightarrow 2(a+b)=(a-b)(a+b) \Leftrightarrow\left[\begin{array}{l}a+b=0 \\a-b=2\end{array}\right.$

  • TH1: $a+b=0 \Leftrightarrow\left\{\begin{array}{l}a=0 \\ b=0\end{array}\right.$ (Loại do $\left.a>0, b>0\right)$

  • TH2: $a-b=2$ khi đó ta có:

$\sqrt{2 x^{2}+x+9}-\sqrt{2 x^{2}-x+1}=2 $

$\Leftrightarrow \sqrt{2 x^{2}+x+9}=2+\sqrt{2 x^{2}-x+1} $

$\Leftrightarrow 2 x^{2}+x+9=4+2 x^{2}-x+1+4 \sqrt{2 x^{2}-x+1} $

$\Leftrightarrow x+2=2 \sqrt{2 x^{2}-x+1}$

$\Leftrightarrow\left\{\begin{array} { l }{ x \geq – 2 } \\{ x ^ { 2 } + 4 x + 4 = 8 x ^ { 2 } – 4 x + 4 }\end{array} \Leftrightarrow \left\{\begin{array}{l}x \geq-2 \\{\left[\begin{array}{l}x=0(n) \\x=\frac{8}{7}(n)\end{array}\right.}\end{array}\right.\right.$

Vậy $S=(0 ; \frac{8}{7})$

b) $\left\{\begin{array}{l}y^{2}-2 x y=8 x^{2}-6 x+1 \\ y^{2}=x^{3}+8 x^{2}-x+1(2)\end{array}\right.$

Từ $(2)$ ta có: $y^{2}=x^{3}+8 x^{2}-x+1$ thay vào $(1)$ ta có: $x^{3}+8 x^{2}-x+1-2 x y=8 x^{2}-6 x+1 \Leftrightarrow x^{3}-2 x y+5 x=0$ $\Leftrightarrow x\left(x^{2}-2 y+5\right)=0 \Leftrightarrow\left[\begin{array}{l}x=0 \\ x^{2}-2 y+5=0\end{array}\right.$

  • TH1: $x=0$ thay vào (2) ta có: $y^{2}=1 \Rightarrow y=\pm 1$

  • TH2: $x^{2}-2 y+5=0 \Leftrightarrow 2 y=x^{2}+5$ thay vào (2) ta có:

$4 y^{2}=4 x^{3}+32 x^{2}-4 x+4 $

$\Leftrightarrow\left(x^{2}+5\right)^{2}=4 x^{3}+32 x^{2}-4 x+4$

$\Leftrightarrow x^{4}-4 x^{3}-22 x^{2}+4 x+21=0 $

$\Leftrightarrow(x-7)(x+3)(x-1)(x+1)=0 $

$\Leftrightarrow\left[\begin{array}{l}x=7 \Rightarrow y=27 \\x=1 \Rightarrow y=3 \\x=-1 \Rightarrow y=3 \\x=-3 \Rightarrow y=7\end{array}\right.$

Vậy nghiệm của hệ phương trình là: $(-3 ; 7),(-1 ; 3),(0 ;-1),(0 ; 1),(1 ; 3)$, $(7 ; 27)$.

Bài 3. Cho tam giác nhọn $A B C(A B<B C<C A)$ nội tiếp đường tròn $(O)$. Từ $A$ kẻ đường thẳng song song với $B C$ cắt $(O)$ tại $A_{1}$. Từ $B$ kẻ đường thẳng song song với $A C$ cắt $(O)$ tại $B_{1}$. Từ $C$ kẻ đường thẳng song song với $A B$ cắt $(O)$ tại $C_{1}$. Chứng minh rằng các đường thẳng qua $A_{1}, B_{1}, C_{1}$ lần lượt vuông góc với $B C, C A, A B$ đồng quy.

Lời giải.

Gọi $H$ là trực tâm của tam giác $A B C$ và $O H$ cắt đường thẳng qua $A_{1}$, vuông góc với $B C$ ở điểm $K$. Gọi $M$ là trung điểm $A A_{1}$ thì $O M \perp A A_{1}$. Suy ra $O M \perp B C$.

Mặt khác, tứ giác $A H K A_{1}$ là hình thang vì $A H | A_{1} K$ nên ta có $O M$ là đường trung bình, kéo theo $O$ là trung điểm $H K$ hay nói cách khác, đường thẳng qua $A_{1}$, vuông góc với $B C$ sẽ đi qua điểm đối xứng với trực tâm $H$ của tam giác $A B C$ qua $O$.

Rõ ràng điểm này bình đẳng với $B, C$ nên hai đường qua $B_{1}, C_{1}$ lần lượt vuông góc với $C A, A B$ cũng đi qua $K$. Vì thế nên ta có các đường thẳng của đề bài đồng quy ở $K$.

Bài 4.

a) Cho 2 số thực $a, b$. Chứng minh rằng: $\frac{a^{2}+b^{2}}{2} \geq a b+\frac{(a-b)^{2}}{a^{2}+b^{2}+2}$

b) Cho hai số dương $a, b$ thỏa mãn điều kiện $a+b \leq 3$ Tìm giá trị nhỏ nhất của biểu thức: $Q=b-a+\frac{20}{a}+\frac{7}{b}$.

Lời giải.

a) Ta có: $\frac{a^{2}+b^{2}}{2} \geq a b+\frac{(a-b)^{2}}{a^{2}+b^{2}+2}$

$\Leftrightarrow \frac{(a-b)^{2}}{2} \geq \frac{(a-b)^{2}}{a^{2}+b^{2}+2} $

$\Leftrightarrow(a-b)^{2}\left(\frac{1}{2}-\frac{1}{a^{2}+b^{2}+2}\right) \geq 0 $

$\Leftrightarrow(a-b)^{2} \frac{a^{2}+b^{2}}{a^{2}+b^{2}+2} \geq 0 (đúng) $

b) Ta có: $a, b>0$ và $a \leq 3-b$

$Q=b-a+\frac{20}{a}+\frac{7}{b} \geq b-(3-b)+\frac{20}{3-b}+\frac{7}{b} $

$=(2 b-3)+\frac{20}{3-b}+\frac{7}{b} $

$=\left[5(3-b)+\frac{20}{3-b}\right]+\left(7 b+\frac{7}{b}\right)-18 $

$\geq 2 \sqrt{100}+2 \sqrt{49}-18=16$

Dấu “=” xảy ra khi và chỉ khi $a=2$ và $b=1$.

Bài 5. Đường tròn $(I)$ nội tiếp tam giác $A B C$ tiếp xúc với các cạnh $A B, B C, C A$ lần lượt tại $D, E, F$. Kẻ đường kính $E J$ của đường tròn $(I)$. Gọi $d$ là đường thẳng qua $A$ song song với $B C$. Đường thẳng $J D$ cắt $d, B C$ lần lượt tại $L, H$.

a) Chứng minh: $E, F, L$ thẳng hàng.

b) $J A, J F$ cắt $B C$ lần lượt tại $M, K$. Chứng minh: $M H=M K$.

Lời giải.

a) Ta có $J E$ là đường kính của $(I)$ nên $\angle J D E=90^{\circ}$ và $\triangle H D E$ vuông ở $D$. Chú ý rằng $B D=B E$, do cùng là tiếp tuyến kẻ từ $B$ đến $(I)$ nên $B D=B H$ (tính chất trung tuyến ứng với cạnh huyền). Do đó $\triangle B H D$ cân ở $B$.

Vị $A L / / B H$ nên $\triangle A D L$ và $\triangle B D H$ đồng dạng, kéo theo $\triangle A D L$ cân ở $A$ hay $A L=A D=A F$.

Vị $A L / / C E$ nên $\angle L A F=\angle F C E$, mà $\triangle A L F, \triangle C E F$ đều cân có các góc ở đỉnh bằng nhau nên chúng đồng dạng.

Suy ra $\angle A F L=\angle C F E$, kéo theo $L, F, E$ thẳng hàng.

b) Kéo dài $J F$ cắt $d$ ở $T$ thì tương tự câu $a$, ta có $T, D, E$ thẳng hàng và $A T=A D=$ $A F=A L$.

Theo định lý Thales với $d / / B C$ thì $\frac{A L}{M H}=\frac{A J}{J M}=\frac{A T}{M K}$, mà $A T=A L$ nên $M H=M K$.

Bài 6. Tìm tất cả các số nguyên dương $x, y$ thỏa mãn phương trình: $3^{x}-y^{3}=1$

Lời giải.

Ta có: $3^{x}-y^{3}=1 \Leftrightarrow y^{3}+1=3^{x}$

$\Leftrightarrow(y+1)\left(y^{2}-y+1\right)=3^{x} \Leftrightarrow\left\{\begin{array}{l}y+1=3^{m}(1) \\ y^{2}-y+1=3^{n}(2)\end{array}\right.$

  • TH1: $m=1 \Leftrightarrow y=2 \Rightarrow x=2$ (nhận).

  • TH2: $m \geq 2 \Leftrightarrow y=3^{m}-1$

Thế vào $(2):\left(3^{m}-1\right)^{2}-\left(3^{m}-1\right)+1=3^{n}$ $\Leftrightarrow 3^{n}=3^{2 m}-3^{m+1}+3=3^{m+1}\left(3^{m-1}-1\right)+3>3^{m+1} \Rightarrow n>m+1$

Ta lại có: $3=3^{n}-3^{2 m}+3^{m+1}=3^{m+1}\left(3^{n-m-1}-3^{m-1}+1\right) \vdots 3^{m+1} \Rightarrow m=0$ (vô lí).

Vậy phương trình có nghiệm $\left\{\begin{array}{l}x=2 \\ y=2\end{array}\right.$.

 

 

 

 

 

 

 

 

 

 

 

 

 

 

 

 

 

 

 

 

 

 

 

 

 

 

 

 

 

 

 

ĐỀ THI VÀO LỚP 10 CHUYÊN TOÁN TP.HCM NĂM 2018

Bài 1. (1 điểm )Cho $a, b, c$ là ba số thỏa điều kiện $a+b+c=0$ và $a^{2}=2(a+c+$ 1) $(a+b-1)$. Tính giá trị của biểu thức $A=a^{2}+b^{2}+c^{2}$.

Bài 2. (2 điểm)

(a) Giải phương trình: $4 \sqrt{x+3}=1+4 x+\frac{2}{x}$.

(b) Giải hệ phương trình: $\left\{\begin{array}{l}x^{2}+y^{3}=1 \\ x^{2}+y^{5}=x^{3}+y^{2}\end{array}\right.$

Bài 3. (2 điểm) Cho tam giác $A B C(A B<A C)$ vuông tại $A$ có đường cao $A H$. Gọi $E, F$ lần lượt là hình chiếu của $H$ lên $A B, A C$.

(a) Chứng minh rằng: $B E \sqrt{C H}+C F \sqrt{B H}=A H \sqrt{B C}$

(b) Gọi $D$ là điểm đối xứng của $B$ qua $H$ và gọi $O$ là trung điểm của $B C$. Đường thẳng đi qua $D$ và vuông với $B C$ cắt $A C$ tại $K$. Chứng minh rằng: $B K$ vuông góc với $A O$.

Bài 4. (1,5 điểm)

(a) Chứng minh rằng: $x^{4}-x+\frac{1}{2}>0$ với mọi số thực $x$.

(b) Cho $x, y$ là các số thực thỏa mãn điều kiện $x^{2}-x y+y^{2}=3$. Tìm giá trị lớn nhất và giá trị nhỏ nhất của biểu thức $P=x^{2}+y^{2}$.

Bài 5. (1,5 điểm) Cho tam giác $A B C$ vuông tại $A$. Gọi $M$ là trung điểm của $B C$ và $O$ là tâm đường tròn ngoại tiếp của tam giác $A M B$. Đường thẳng $A C$ cắt $(O)$ tại điểm thứ hai $K$. Đường thẳng $B K$ cắt đường tròn ngoại tam giác $A B C$ tại $L$. Các đường thẳng $C L$ và $K M$ cắt nhau tại $E$. Chứng minh rằng $E$ nằm trên đường tròn ngoại tiếp tam giác $A C M$.

Bài 6. (2 điểm) Các số nguyên dương từ 1 đến 2018 được tô màu theo nguyên tắc sau: Các số mà khi chia hết cho 24 dư 17 tô được màu xanh. Các số mà khi chia cho 40 dư 7 được tô màu đỏ. Các số còn lại được tô màu vàng.

(a) Chứng tỏ rằng không có số nào được tô cả hai màu xanh và đỏ. Hỏi có bao nhiêu số được tô màu vàng?

(b) Có bao nhiêu cặp số $(a, b)$ sao cho $a$ được tô màu xanh, $b$ được tô màu đỏ và $|a-b|$ bằng 2 ?

LỜI GIẢI

 

Bài 1. Cho $a, b, c$ là ba số thỏa điều kiện $a+b+c=0$ và $a^{2}=2(a+c+1)(a+b-1)$. Tính giá trị của biểu thức $A=a^{2}+b^{2}+c^{2}$.

Lời giải. Ta có $: a+b+c=0 \Rightarrow\left\{\begin{array}{l}a+c=-b \\ a+b=-c \\ a=-(b+c)\end{array}\right.$

Khi đó: $a^{2}=2(a+c+1)(a+b-1)=2(-b+1)(-c-1)$ $\Leftrightarrow a^{2}=2(b-1)(c+1)$

$\Leftrightarrow a^{2}=2(b c+b-c-1)$

$\Leftrightarrow(b+c)^{2}=2(b c+b-c-1)$

$\Leftrightarrow b^{2}+2 b c+c^{2}=2 b c+2 b-2 c-2$

$\Leftrightarrow(b-1)^{2}+(c+1)^{2}=0 \Leftrightarrow\left\{\begin{array}{l}b=1 \\ c=-1 .\end{array}\right.$

Suy ra $a=0$.

Vậy $A=a^{2}+b^{2}+c^{2}=0^{2}+1^{2}+(-1)^{2}=2$.

Bài 2.

a) Giải phương trình: $4 \sqrt{x+3}=1+4 x+\frac{2}{x}$.

b) Giải hệ phương trình: $\left\{\begin{array}{l}x^{2}+y^{3}=1 \\ x^{2}+y^{5}=x^{3}+y^{2}\end{array}\right.$

Lời giải.

a) Giải phương trình: $4 \sqrt{x+3}=1+4 x+\frac{2}{x}$.

Điều kiện xác định: $\left\{\begin{array}{l}x \neq 0 \\ x \geq 3 .\end{array}\right.$

Ta có: $(1) \Leftrightarrow 4 x \sqrt{x+3}=x+4 x^{2}+2 $

$\Leftrightarrow 4 x^{2}-4 x \sqrt{x+3}+(x+3)=1 $

$\Leftrightarrow(2 x-\sqrt{x+3})^{2}=1 $

$\Leftrightarrow\left[\begin{array}{l}2 x-\sqrt{x+3}=1 \\2 x-\sqrt{x+3}=-1\end{array}\right.$

– Trường hợp 1: $2 x-1=\sqrt{x+3} \Leftrightarrow\left\{\begin{array}{l}2 x-1 \geq 0 \\ (2 x-1)^{2}=x+3\end{array} \Leftrightarrow\left\{\begin{array}{l}x \geq \frac{1}{2} \\ 4 x^{2}-5 x-2=0\end{array}\right.\right.$

– Trường hợp 2: $2 x+1=\sqrt{x+3} \Leftrightarrow\left\{\begin{array}{l}2 x+1 \geq 0 \\ (2 x+1)^{2}=x+3\end{array} \Leftrightarrow\left\{\begin{array}{l}x \geq-\frac{1}{2} \\ 4 x^{2}+3 x-2=0\end{array}\right.\right.$

$\Leftrightarrow\left\{\begin{array}{l}x \geq-\frac{1}{2} \\{\left[\begin{array}{l}x=\frac{-3+\sqrt{41}}{8} \\x=\frac{-3-\sqrt{41}}{8}\end{array} \Rightarrow x=\frac{-3+\sqrt{41}}{8}\right.}\end{array}\right.$

Vậy tập nghiệm của phương trình đã cho là $\mathcal{S}=(\frac{5+\sqrt{57}}{8} ; \frac{-3+\sqrt{41}}{8})$

b) $\left\{\begin{array}{l}x^{2}+y^{3}=1 \\x^{2}+y^{5}=x^{3}+y^{2}\end{array}\right.\quad(2)$

Ta có phương trình

$(2) \Leftrightarrow x^{2}(1-x)=y^{2}\left(1-y^{3}\right)=y^{2} x^{2} \Leftrightarrow x^{2}\left(1-x-y^{2}\right)=0 \Leftrightarrow\left[\begin{array}{l}x=0 \\ 1-x-y^{2}=0\end{array}\right.$

– Trường hợp 1: Với $x=0$ thì $(1) \Leftrightarrow y^{3}=1 \Leftrightarrow y=1$.

– Trường hợp 2: Với $1-x-y^{2}=0 \Leftrightarrow x=1-y^{2}$ thì

(1) $\Leftrightarrow\left(1-y^{2}\right)^{2}+y^{3}=1 $

$\Leftrightarrow y^{4}+y^{3}-2 y^{2}=0 $

$\Leftrightarrow y^{2}\left(y^{2}+y-2\right)=0 $

$\Leftrightarrow y^{2}(y-1)(y+2)=0 $

$\Leftrightarrow\left[\begin{array}{l}y=0 \\\\y=1 \\\\y=-2 \end{array}\right. $

  • Với $y=0$ thì $x=1$.

  • Với $y=1$ thì $x=0$.

  • Với $y=-2$ thì $x=-3$.

Vậy hệ phương trình đã cho có nghiệm là: $\mathcal{S}={(0 ; 1),(1 ; 0),(-3 ;-2)}$

Bài 3. Cho tam giác $A B C(A B<A C)$ vuông tại $A$ có đường cao $A H$. Gọi $E, F$ lần lượt là hình chiếu của $H$ lên $A B, A C$.

a) Chứng minh rằng: $B E \sqrt{C H}+C F \sqrt{B H}=A H \sqrt{B C}$

b) Gọi $D$ là điểm đối xứng của $B$ qua $H$ và gọi $O$ là trung điểm của $B C$. Đường thẳng đi qua $D$ và vuông với $B C$ cắt $A C$ tại $K$. Chứng minh rằng: $B K$ vuông góc với $A O$.

Lời Giải.

a) Chứng minh rằng: $B E \sqrt{C H}+C F \sqrt{B H}=A H \sqrt{B C}$.

Ta có: $H E | A C$ (vì cùng vuông góc với $A B$ ).

Suy ra: $\frac{B E}{A B}=\frac{B H}{B C} \quad$ (1) (theo định lí Ta- lét).

Ta lại có: $H F | A B$ (Vi cùng vuông góc với $A C$ ).

Suy ra: $\frac{C F}{A C}=\frac{C H}{B C} \quad$ (2) (theo định lí Ta-lét).

Từ (1) và (2) suy ra  $\frac{B E}{A B}+\frac{C F}{A C}=\frac{B H}{B C}+\frac{C H}{B C}=1 $

$\Rightarrow B E \cdot A C+C F \cdot A B=A B \cdot A C $

$\Leftrightarrow B E \cdot \sqrt{C H \cdot B C}+C F \sqrt{B H \cdot B C}=A H \cdot B C $

Vì $ A H \cdot B C=A B \cdot A C) $

$\Leftrightarrow \sqrt{B C}(B E \sqrt{C H}+C F \sqrt{B H})=\sqrt{B C} \cdot A H \cdot \sqrt{B C} $

$\Leftrightarrow B E \sqrt{C H}+C F \sqrt{B H}=A H \sqrt{B C} . \text { (dpcm) }$

b) Ta có: $\triangle C D K \sim \triangle C A B \quad(g-g)$

Suy ra: $\frac{C D}{C K}=\frac{C A}{C B}$

Xét $\triangle A C D$ và $\triangle B C K$ có:

$C$ chung và $\frac{C D}{C K}=\frac{C A}{C B}(\mathrm{cmt})$.

Suy ra: $\triangle A C D \sim \triangle B C K \quad(c-g-c)$.

Do đó: $\widehat{K B D}=\widehat{C A D}=90^{\circ}-\widehat{B A D}=90^{\circ}-2 \widehat{B A H}=90^{\circ}-2 \widehat{B C A}($ Vi $\widehat{B A H}=$ $\widehat{B C A}$ (cùng phụ với $A B C$) (1)

Mà $\triangle A B C$ vuông tại $A$ có $O$ là trung điểm của $B C$.

Suy ra $O$ là tâm đường tròn ngoại tiếp $\triangle A B C \Rightarrow 2 \widehat{A C B}=\widehat{A O B}$ Khi đó $(1) \Leftrightarrow \widehat{K B D}=90^{\circ}-\widehat{A O B}$.

Vậy $B K \perp A O$ (đpcm).

Bài 4.

a) Chứng minh rằng: $x^{4}-x+\frac{1}{2}>0$ với mọi số thực $x$.

b) Cho $x, y$ là các số thực thỏa mãn điều kiện $x^{2}-x y+y^{2}=3$. Tìm giá trị lớn nhất và giá trị nhỏ nhất của biểu thức $P=x^{2}+y^{2}$.

Lời giải.

a) Ta có: $x^{4}-x+\frac{1}{2}=x^{4}-x^{2}+x^{2}-x+\frac{1}{2}=x^{4}-2 \cdot \frac{1}{2} x^{2}+\frac{1}{4}+x^{2}-2 \cdot \frac{1}{2} x+\frac{1}{4}$ $=\left(x^{2}-\frac{1}{2}\right)^{2}+\left(x-\frac{1}{2}\right)^{2} \geq 0$ với mọi số thực $x$.

Dấu bằng xảy $\mathrm{ra} \Leftrightarrow\left\{\begin{array}{l}x^{2}-\frac{1}{2}=0 \\x-\frac{1}{2}=0\end{array} \Leftrightarrow\left\{\begin{array}{l}x^{2}=\frac{1}{2} \\x=\frac{1}{2}\end{array}\right.\right.$ Hệ phương trình vô nghiệm.

Do đó đẳng thức không xảy ra, tức là $x^{4}-x+12>0$ với mọi số thực $x$.

b) Cách 1:

Ta có: $A=\frac{P}{3}=\frac{x^{2}+y^{2}}{x^{2}-x y+y^{2}}$

Xét $y=0 \Rightarrow x=\pm \sqrt{3} \Rightarrow P=3$.

Xét $y \neq 0$, khi đó $A=\frac{\left(\frac{x}{y}\right)^{2}+1}{\left(\frac{x}{y}\right)^{2}-\frac{x}{y}+1} \quad(1)$. Đặt $t=\frac{x}{y}$.

Khi đó $(1)$ trở thành $A=\frac{t^{2}+1}{t^{2}-t+1} \Leftrightarrow t^{2}(A-1)+A t+A-1=0$.

Xét $A=1 \Rightarrow P=3$.

Xét $A \neq 1 \Rightarrow \Delta=A^{2}-4(A-1)^{2}=-(3 A-2)(A-2) \geq 0 \Leftrightarrow \frac{2}{3} \leq A \leq 2 \Leftrightarrow$ $2 \leq P \leq 6$.

$\operatorname{Min} P=2$ khi $x=1 ; y=-1$ hoặc $x=-1 ; y=1$.

$\operatorname{Max} P=6$ khi $x=y=\pm \sqrt{3}$.

Cách 2: $3=\frac{1}{2}\left(2 x^{2}-2 x y+2 y^{2}\right)=\frac{1}{2}\left[3\left(x^{2}+y^{2}\right)-(x-y)^{2}\right]$.

$\Rightarrow x^{2}+y^{2} \geq 2$

$3=\frac{1}{2}\left(2 x^{2}-2 x y+2 y^{2}\right)=\frac{1}{2}\left[x^{2}+y^{2}+(x-y)^{2}\right] .$

$\Rightarrow x^{2}+y^{2} \leq 6$

Vậy $2 \leq P \leq 6$.

Bài 5. Cho tam giác $A B C$ vuông tại $A$. Gọi $M$ là trung điểm của $B C$ và $O$ là tâm đường tròn ngoại tiếp của tam giác $A M B$. Đường thẳng $A C$ cắt $(O)$ tại điểm thứ hai $K$. Đường thẳng $B K$ cắt đường tròn ngoại tam giác $A B C$ tại $L$. Các đường thẳng $C L$ và $K M$ cắt nhau tại $E$. Chứng minh rằng $E$ nằm trên đường tròn ngoại tiếp tam giác ACM.

Lời giải. Ta có: tứ giác $A K M B$ nội tiếp ( tổng hai góc đối bằng $180^{\circ}$ )

Suy ra: $\widehat{M A K}=\widehat{K B M}=\widehat{L B C}$ (cùng chắn cung $K M$ )

Ta lại có: $\widehat{L B C}=\widehat{M E C}$ (cùng phụ với $\widehat{E C B}$ ) $\Rightarrow \widehat{M A K}=\widehat{M E C}$

Suy ra tứ giác $M A E C$ nội tiếp

Vậy $E$ nằm trên đường tròn ngoại tiếp $\triangle A M C$.

Bài 6. Các số nguyên dương từ 1 đến 2018 được tô màu theo nguyên tắc sau: Các số mà khi chia hết cho 24 dư 17 tô được màu xanh. Các số mà khi chia cho 40 dư 7 được tô màu đỏ. Các số còn lại được tô màu vàng.

a) Chứng tỏ rằng không có số nào được tô cả hai màu xanh và đỏ. Hỏi có bao nhiêu số được tô màu vàng?

b) Có bao nhiêu cặp số $(a, b)$ sao cho $a$ được tô màu xanh, $b$ được tô màu đỏ và $|a-b|$ bằng 2 ?

Lời giải.

a) Theo đề bài ta có

  • Số màu xanh có dạng: $24 x+17 \quad(x \in \mathbb{N})$ với $1 \leq 24 x+17 \leq 2018 \Leftrightarrow-\frac{2}{3} \leq x \leq \frac{667}{8} \Leftrightarrow 0 \leq x \leq 83 .$

Do đó có 84 số được tô màu xanh.

  • Số màu đỏ có dạng: 40y $+7 \quad(y \in \mathbb{N})$ với $1 \leq 40 y+7 \leq 2018 \Leftrightarrow-\frac{3}{20} \leq x \leq \frac{2011}{40} \Leftrightarrow 0 \leq x \leq 50$.

Do đó có 51 số được tô màu đỏ.

  • Giả sử có số được tô cả hai màu xanh và đỏ khi đó tồn tại $x_{0} ; y_{0}$ sao cho $24 x_{0}+17=40_{0} y+7 \Leftrightarrow 24_{0} x+10=40_{0} y$

Vì $24: 4$ và $40: 4$ nên $10: 4$ (vô lí)

Vậy không có số nào được tô cả hai màu xanh và đỏ. Khi đó số lượng số được tô màu vàng là: $2018-84-51=1883$ (số).

b) Ta có $a=24 x+17$ và $b=40 y+7$ với $(x, y \in \mathbb{N})$ Xét 2 trường hợp sau:

  • Trường hợp 1: $a-b=2 \Leftrightarrow 24 x-40 y+10=2 \Leftrightarrow 3 x-5 y=-1 \Leftrightarrow 3 x=$ $5 y-1 .$

Mà $0 \leq x \leq 83 \Leftrightarrow 0 \leq 3 x \leq 249$

Suy ra $0 \leq 5 y-1 \leq 249 \Leftrightarrow \frac{1}{5} \leq y \leq 50$.

Vi $(5 y-1)$ : 3 nên $y$ chia 3 dư 2 .

Kiểm tra ta thấy $y$ nhận 17 giá trị khác nhau $\Rightarrow$ có 17 cặp $(x ; y)$.

  • Trường hợp 2: $a-b=-2 \Leftrightarrow 24 x+17-40 y-7=-2 \Leftrightarrow 24 x-40 y=$ $-12$

$\Leftrightarrow 6 x-10 y=-3$ (loại)

Vậy có 17 cặp $(a ; b)$ thỏa mãn yêu cầu bài toán.

 

 

 

 

 

 

 

 

 

 

 

 

 

 

 

 

 

 

 

 

 

 

 

 

 

 

 

 

 

 

 

 

 

 

 

 

 

 

 

 

ĐỀ THI VÀO LỚP 10 CHUYÊN TOÁN TP.HCM NĂM 2019

Bài 1. Cho $a, b, c$ là ba số thực thỏa mãn điều kiện $a+b+c=1$. Tính giá trị của biểu thức

$A=a^{3}+b^{3}+c^{3}-3(a b+c)(c-1)$

Bài 2. (a) Giải phương trình:

$5 \sqrt{x-1}-\sqrt{x+7}=3 x-4$

(b) Giải hệ phương trình:

$\left\{\begin{array}{l}2(x+y)-x y=4 \\x y(x+y-4)=-2\end{array}\right.$

Bài 3. Đường tròn nội tiếp tam giác $A B C$ tiếp xúc với các cạnh $B C, C A, A B$ lần lượt tại $M, N, P$. Gọi $K$ là hình chiếu vuông góc của $M$ lên $N P$. Chứng minh rằng $K M$ là phân giác của góc $\angle B K C$.

Bài 4. Cho $x, y, z$ là các số thực thuộc đoạn $[0,2]$ thỏa mãn điều kiện $x+y+z=3$.

(a) Chứng minh rằng

$x^{2}+y^{2}+z^{2}<6$

(b) Tìm giá trị lớn nhất của biểu thức

$P=x^{3}+y^{3}+z^{3}-3 x y z$

Bài 5. Cho tam giác đều $A B C$. Gọi $M, N$ là hai điểm nằm trên cạnh $B C$ sao cho $\angle M A N=30^{\circ}(M$ nằm giữa $B$ và $N)$. Gọi $K$ là giao điểm của hai đường tròn $(A B N)$ và $(A C M)(K$ khác $A)$. Chứng minh rằng:

(a) Hai điểm $K$ và $C$ đối xứng với nhau qua $A N$.

(b) Đường thẳng $A K$ đi qua tâm đường tròn $(A M N)$.

Bài 6. Cho $m, n$ là hai số nguyên. Chứng minh rằng, nếu $7(m+n)^{2}+2 m n$ chia hết cho 225 thì $m n$ cũng chia hêt cho 225 .

 

LỜI GIẢI

 

Bài 1.Cho $a, b, c$ là ba số thực thỏa mãn điều kiện $a+b+c=1$. Tính giá trị của biểu thức

$A=a^{3}+b^{3}+c^{3}-3(a b+c)(c-1)$

Lời giải. $A=(a+b)^{3}-3 a b(a+b)+c^{3}+3(a b+c)(a+b)$

$=(a+b)^{3}+c^{3}+3(a+b) c $

$=(a+b)^{3}+c^{3}+3(a+b) c(a+b+c) $

$=(a+b+c)^{3}=1$

Bài 2.

a) Giải phương trình:

$5 \sqrt{x-1}-\sqrt{x+7}=3 x-4$

b) Giải hệ phương trình:

$\left\{\begin{array}{l}2(x+y)-x y=4 \\ x y(x+y-4)=-2\end{array}\right.$

Lời giải.

a) Điều kiện $x \geq 1.5 \sqrt{x-1}-\sqrt{x+7}=3 x-4$

$\Leftrightarrow \frac{25(x-1)-(x+7)}{5 \sqrt{x-1}+\sqrt{x+7}}=3 x-4 $

$\Leftrightarrow \frac{8(3 x-4)}{5 \sqrt{x-1}+\sqrt{x+7}}=3 x-4 $

$3 x-4=0$ (1) hoặc $5 \sqrt{x-1}+\sqrt{x+7}=8(2) $

$(1) \Leftrightarrow x=\frac{4}{3}(\text { nhận }) $

$(2)  64=25(x-1)+x+7+10 \sqrt{(x-1)(x+7)} $

$\Leftrightarrow 82-26 x=10 \sqrt{\left(x^{2}+6 x-7\right)}$

Giải ra được nghiệm $x=2$.

Vậy phương trình có hai nghiệm $S=(2, \frac{4}{3})$.

b) Từ phương trình (1) ta có $(x-2)(y-2)=0 \Leftrightarrow x=2$ hoặc $y=2$. Với $x=2$ thế vào $(2)$ ta có $y=1$. Ta có nghiệm $(x ; y)$ là $(2 ; 1)$.

Với $y=2$ thế vào $(2)$ ta có $y=1$. Ta có nghiệm $(x ; y)$ là $(1 ; 2)$.

Vậy hệ phương trình có hai nghiệm $(x ; y)$ là $(2 ; 1)$ và $(1 ; 2)$.

Bài 3. Đường tròn nội tiếp tam giác $A B C$ tiếp xúc với các cạnh $B C, C A, A B$ lần lượt tại $M, N, P$. Gọi $K$ là hình chiếu vuông góc của $M$ lên $N P$. Chứng minh rằng $K M$ là phân giác của góc $\angle B K C$.

Lời giải. Vẽ $B X, C Y$ vuông góc với $P N$ tại $X, Y$. Ta có $\angle A P=A N$ nên tam giác $A P N$ cân.

Suy ra $\angle A P N=\angle A N P ;$ mà $\angle B P X=\angle A P N, \angle C N Y=\angle A N P$ nên $\angle B P X=\angle C N Y$. Do đó $\triangle B P X \backsim \triangle C N Y$, suy ra $\frac{B X}{C Y}=\frac{B P}{C N}$.

Mà $B P=B M, C N=C M$ suy ra $\frac{B P}{C N}=\frac{B M}{C M}=\frac{X K}{Y K}$.

Do đó $\frac{B X}{C Y}=\frac{X K}{Y K}$.

suy ra $\triangle B X K \backsim \triangle C Y K$ do đó $\angle X K B=\angle C K Y$ mà $M K \perp X Y$ nên $K M$ là phân giác $\angle B K C$.

Bài 4.Cho $x, y, z$ là các số thực thuộc đoạn $[0,2]$ thỏa mãn điều kiện $x+y+z=3$.

a) Chứng minh rằng

$x^{2}+y^{2}+z^{2}<6$

b) Tìm giá trị lớn nhất của biểu thức

$P=x^{3}+y^{3}+z^{3}-3 x y z$

Lời giải.

a) Ta có $x, y, z \in[0 ; 2]$ nên $x(2-x) \geq 0 \Leftrightarrow x^{2} \leq 2 x$, tương tự $y^{2} \leq 2 y$, $z^{2} \leq 2 z$. Suy ra $x^{2}+y^{2}+z^{2} \leq 2(x+y+z)=6$. Đẳng thức xảy ra khi $x=0$ hoặc $x=2$, $y=0$ hoặc $y=2, z=0$ hoặc $z=2$ và $x+y+z=3$ (vô nghiệm).

Vậy $x^{2}+y^{2}+z^{2}<6$.

b) $x^{3}+y^{3}+z^{3}-3 x y z=(x+y+z)\left(x^{2}+y^{2}+z^{2}-x y-z y-z x\right)=3\left(x^{2}+y^{2}+\right.$ $\left.z^{2}\right)-\frac{3}{2}\left((x+y+z)^{2}-x^{2}-y^{2}-z^{2}\right)=\frac{9}{2}\left(x^{2}+y^{2}+z^{2}\right)-\frac{27}{2}$.

Không mất tính tổng quát, giả sử $z=\max x, y, z$, suy ra $z \geq 1$.

Ta có $x^{2}+y^{2}+z^{2}=(x+y)^{2}+z^{2}-2 x y=(3-z)^{2}+z^{2}-2 x y=2 z^{2}-6 z+$ $9-2 x y=2(z-1)(z-2)-2 x y+5 \leq 5$.

Đẳng thức xảy ra khi $z=2, x=0, y=1$.

Do đó $x^{3}+y^{3}+z^{3}-3 x y z \leq 9$, đẳng thức xảy ra khi $z=2, x=0, y=1$. Vậy giá trị lớn nhất của biểu thức $x^{3}+y^{3}+z^{3}-3 x y z$ là 9 .

Bài 5. Cho tam giác đều $A B C$. Gọi $M, N$ là hai điểm nằm trên cạnh $B C$ sao cho $\angle M A N=30^{\circ}(M$ nằm giữa $B$ và $N)$. Gọi $K$ là giao điểm của hai đường tròn $(A B N)$ và $(A C M)(K$ khác $A)$. Chứng minh rằng:

a) Hai điểm $K$ và $C$ đối xứng với nhau qua $A N$.

b) Đường thẳng $A K$ đi qua tâm đường tròn $(A M N)$.

Lời giải.

a) Gọi $K$ là điểm đối xứng của $C$ qua $A N$. Có

$\angle A K^{\prime} N=\angle A C N=\angle A B N$

nên tứ giác $A B K^{\prime} N$ nội tiếp. Suy ra $K^{\prime} \in(A B N)$. Có

$\angle M A K^{\prime}+\angle N A C=\angle M A K^{\prime}+\angle K^{\prime} A N=30^{\circ}$

$\angle B A M+\angle N A C=30^{\circ}$

suy ra $\angle M A K^{\prime}=\angle B A M$.

Suy ra $\triangle A B M=\triangle A K^{\prime} M(c-g-c)$ nên $\angle A K^{\prime} M=\angle A B C=\angle A C B$ ta thu được $K^{\prime} \in(A M C)$. Vậy $K \equiv K^{\prime}$ ta có điều phải chứng minh.

b) Gọi $O$ là tâm $(A M N)$.

Có $\angle M K A=\angle M C A=\angle A K N=60^{\circ}$ nên $\angle M K N=120^{\circ}$. Mà $\angle M O N=$ $2 \angle M A N=60^{\circ}$ nên tứ giác $M O N K$ nội tiếp.

Lại có $O M=O N$ nên $\angle O K N=\angle O K M=60^{\circ}$ và $\angle A K N=60^{\circ}$ nên $A, O, K$ thẳng hàng.

Bài 6. Cho $m, n$ là hai số nguyên. Chứng minh rằng, nếu $7(m+n)^{2}+2 m n$ chia hết cho 225 thì mn cũng chia hết cho 225 .

Lời giải. Đặt $A=7(m+n)^{2}+2 m n$, ta có $2 A=14(m+n)^{2}+4 m n=15(m+n)^{2}-(m-$ $n)^{2}$ chia hết cho 225 , suy ra $(m-n)^{2}$ chia hết cho 15 .

Ta có $(m-n)^{2}$ chia hết cho 3,5 suy ra $m-n$ chia hết cho 3 và 5 (do 3,5 là số nguyên tố), do đó $m-n$ chia hết cho 15 , suy ra $(m-n)^{2}$ chia hết cho 225 .

Khi đó $15(m+n)^{2}$ chia hết cho 225 , suy ra $(m+n)^{2}$ chia hết cho 15 , tương tự trên thì $(m+n)^{2}$ chia hết cho 225 .

Khi đó $4 m n=(m+n)^{2}-(m-n)^{2}$ chia hết cho 225 , mà $(4,225)=1$ nên $m n$ chia hết cho $225 .$

 

 

 

 

 

 

 

 

 

 

 

 

 

 

 

 

 

 

 

 

 

 

 

 

 

 

 

 

 

 

 

 

 

 

 

 

ĐỀ THI VÀO LỚP 10 CHUYÊN TOÁN TP.HCM NĂM 2017

Bài 1. (a) Cho các số thực $a, b, c$ sao cho $a+b+c=3, a^{2}+b^{2}+c^{2}=29$ và $a b c=11$. Tính $a^{5}+b^{5}+c^{5}$

(b) Cho biểu thức $A=(m+n)^{2}+3 m+n$ với $m, n$ là các số nguyên dương. Chứng minh rằng nếu $A$ là một số chính phương thì $n^{3}+1$ chia hết cho $m$.

Bài 2. (a) Giải hệ phương trình: $2(x+2) \sqrt{3 x-1}=3 x^{2}-7 x-3$

(b) Giải hệ phương trình: $\left\{\begin{array}{l}x+\frac{1}{y}-\frac{10}{x}=-1 \\ 20 y^{2}-x y-y=1\end{array}\right.$

Bài 3. Cho tam giác $A B C$ có $A B<A C<B C$. Trên các cạnh $B C, A C$ lần lượt lấy các điểm $M, N$ sao cho $A N=A B=B M$. Các đường thẳng $A M$ và $B N$ cắt nhau tại $\mathrm{K}$. Gọi $H$ là hình chiếu của $K$ lên $\mathrm{AB}$. Chứng minh rằng:

(a) Tâm đường tròn nội tiếp tam giác $A B C$ nằm trên $K H$.

(b) Các đường tròn nội tiếp các tam giác $A C H$ và $B C H$ tiếp xúc với nhau.

Bài 4. Cho $x, y$ là 2 số thực dương. Tìm giá trị nhỏ nhất của biếu thức:

$P=\frac{16 \sqrt{x y}}{x+y}+\frac{x^{2}+y^{2}}{x y}$

Bài 5. Cho tam giác $A B C$ có góc $B$ tù. Đường tròn $(O)$ nội tiếp tam giác $A B C$ tiếp xúc với các cạnh $A B, C A, B C$ lần lượt tại $L, H, J$.

(a) Các tia $B O, C O$ cắt $L H$ lần lượt tại $M, N$. Chứng minh 4 điểm $B, C, M, N$ cùng thuộc một đường tròn.

(b) Gọi $d$ là đường thẳng qua $O$ và vuông góc với $A J$; $d$ cắt $A J$ và đường trung trực của cạnh $B C$ lần lượt tại $D$ và $F$. Chứng minh 4 điểm $B, D, F, C$ cùng thuộc một đường tròn.

Bài 6. Trên một đường tròn có 9 điểm phân biệt, các điểm này được nối với nhau bởi các đoạn thẳng màu xanh hoặc màu đỏ. Biết rằng mỗi tam giác tạo bởi 3 trong 9 điểm chứa ít nhất một cạnh màu đỏ. Chứng minh rằng tồn tại 4 điểm sao cho 6 đoạn thẳng nối chúng đều có màu đỏ.

LỜI GIẢI

Bài 1.

a) Cho các số thực $a, b, c$ sao cho $a+b+c=3, a^{2}+b^{2}+c^{2}=29$ và $a b c=11$. Tính $a^{5}+b^{5}+c^{5}$

b) Cho biểu thức $A=(m+n)^{2}+3 m+n$ với $m, n$ là các số nguyên dương. Chứng minh rằng nếu $A$ là một số chính phương thì $n^{3}+1$ chia hết cho $m$.

Lời giải.

a) Đặt $S_{2}=a^{2} b^{2}+b^{2} c^{2}+a^{2} c^{2} ; S_{3}=a^{3}+b^{3}+c^{3} ; S_{5}=a^{5}+b^{5}+c^{5}$

  • $(a+b+c)^{2}=a^{2}+b^{2}+c^{2}+2(a b+b c+a c) $

$\quad\quad\Rightarrow 9=29+2(a b+b c+a c) $

$\quad\quad\Rightarrow a b+b c+a c=-10 $

  • $(a+b+c)\left(a^{2}+b^{2}+c^{2}\right)=a^{3}+b^{3}+c^{3}+a^{2} b+a b^{2}+b^{2} c+b c^{2}+a^{2} c+a c^{2} $

$\quad\quad\Rightarrow 3.29=S_{3}+a b(a+b)+b c(b+c)+a c(a+c) $

$\quad\quad\Rightarrow 87=S_{3}+(a+b+c)(a b+b c+a c)-3 a b c $

$\quad\quad \Rightarrow S_{3}=87-3 \cdot(-10)+3.11=150 $

  • $(a b+b c+a c)^{2}=a^{2} b^{2}+b^{2} c^{2}+a^{2} c^{2}+2 a b c(a+b+c) $

$\quad\quad \Rightarrow 100=S_{2}+2.11 .3 \Rightarrow S_{2}=34 $

  • $\left(a^{2}+b^{2}+c^{2}\right)\left(a^{3}+b^{3}+c^{3}\right)=a^{5}+b^{5}+c^{5}+a^{2} b^{3}+a^{3} b^{2}+b^{2}c^{3}+b^{3}c^{2}+a^{3} c^{2}+a^{2} c^{3} $

$\quad\quad\Rightarrow 29.150=S_{5}+a^{2} b^{2}(a+b)+b^{2} c^{2}(b+c)+a^{2} c^{2}(a+c) $

$\quad\quad \Rightarrow 29.150=S_{5}+(a+b+c)\left(a^{2} b^{2}+b^{2} c^{2}+a^{2} c^{2}\right)-a b c(a b+b c+a c) $

$\quad\quad\Rightarrow S_{5}=29.150-3.34+11 .(-10)=4138$

Nhận xét thêm: Trên thực tế, phương trình bậc 3 nhận $a, b, c$ làm nghiệm chỉ có một nghiệm thực, mà đề thi cho 3 số $a, b, c$ thực.

b) Do $m, n$ là số nguyên dương nên ta có: $A=(m+n)^{2}+3 m+n>(m+n)^{2}$ $A=(m+n)^{2}+3 m+n=m^{2}+n^{2}+3 m+n+2 m n<m^{2}+n^{2}+4+4 m+4 n+$ $2 m n=(m+n+2)^{2}$

Mà $\mathrm{A}$ là số chính phương nên $A=(m+n+1)^{2}$

$\Rightarrow(m+n)^{2}+3 m+n=(m+n+1)^{2} $

$\Rightarrow m^{2}+n^{2}+2 m n+3 m+n=m^{2}+n^{2}+1+2 m+2 n+2 m n $

$\Rightarrow m=n+1 .$

Lại có $n^{3}+1=(n+1)\left(n^{2}-n+1\right) \vdots(n+1) \Rightarrow n^{3}+1 \vdots m$

Bài 2.

a) Giải hệ phương trình: $2(x+2) \sqrt{3 x-1}=3 x^{2}-7 x-3$

b) Giải hệ phương trình: $\left\{\begin{array}{l}x+\frac{1}{y}-\frac{10}{x}=-1 \\ 20 y^{2}-x y-y=1\end{array}\right.$

Lời giải.

a) Điều kiện: $x \geq \frac{1}{3}$

$2(x+2) \sqrt{3 x-1}=3 x^{2}-7 x-3 $

$\Leftrightarrow x^{2}+4 x+4+2(x+2) \sqrt{3 x-1}+3 x-1=4 x^{2} $

$\Leftrightarrow(x+2+\sqrt{3 x-1})^{2}=(2 x)^{2} $

$\Leftrightarrow x+2+\sqrt{3 x-1}=2 x \quad\left(\text { vì } x \geq \frac{1}{3}\right) $

$\Leftrightarrow \sqrt{3 x-1}=x-2(x \geq 2) $

$\Leftrightarrow 3 x-1=x^{2}-4 x+4 $

$\Leftrightarrow x^{2}-7 x+5=0\Leftrightarrow\left[\begin{array}{rl}x & =\frac{7+\sqrt{29}}{2}(n) \\ x & =\frac{7-\sqrt{29}}{2}(l)\end{array}\right.$

$\left\{\begin{array}{l}x+\frac{1}{y}-\frac{10}{x}=-1 \\ 20 y^{2}-x y-y=1\end{array}\right.$

Điều kiện: $y \neq 0, x \neq 0$

Chia 2 vế của (2) cho $y$ ta được:

$20 y-x-1=\frac{1}{y}$

$\Rightarrow 20 y-x=\frac{1}{y}+1$

Mà $\frac{1}{y}+1=\frac{10}{x}-x$ nên

$20 y-x=\frac{10}{x}-x$

$\Rightarrow x y=\frac{1}{2}$

Thay vào (2) ta được:

$20 y^{2}-\frac{1}{2}-y=1 \Rightarrow 40 y^{2}-2 y-3=0 \Rightarrow\left[\begin{array}{l}y=\frac{3}{10} \Rightarrow x=\frac{5}{3} \\ y=-\frac{1}{4} \Rightarrow x=-2\end{array}\right.$

Vậy nghiệm của hệ phương trình là $(x ; y) \in[(-2 ;-\frac{1}{4}),(\frac{5}{3} ; \frac{3}{10})]$

Bài 3. Cho tam giác $A B C$ có $A B<A C<B C$. Trên các cạnh $B C, A C$ lần lượt lấy các điểm $M, N$ sao cho $A N=A B=B M$. Các đường thẳng $A M$ và $B N$ cắt nhau tại $\mathrm{K}$. Gọi $H$ là hình chiếu của $K$ lên $\mathrm{AB}$. Chứng minh rằng:

a) Tâm đường tròn nội tiếp tam giác $A B C$ nằm trên $K H$.

b) Các đường tròn nội tiếp các tam giác $A C H$ và $B C H$ tiếp xúc với nhau.

Lời giải.

a) Gọi $I$ là tâm đường tròn nội tiếp tam giác $A B C$.

Tam giác $A B N$ cân tại $A$ nên phân giác góc $B A C$ cũng là đường cao, suy ra $A I \perp B N$.

Tam giác $A B M$ cân tại $B$ nên phân giác góc $A B C$ cũng là đường cao, suy ra $B I \perp A M$.

Suy ra $I$ là trực tâm tam giác $A B K$, mà $K H \perp A B$ nên $K, I, H$ thẳng hàng.

Vậy tâm đường tròn nội tiếp tam giác $A B C$ nằm trên $K H$.

b) Gọi $D$ là tiếp điểm của $(J)$ với $C H$. TA có $D H=\frac{H A+H C-A C}{2}$.

Gọi $E$ là tiếp điểm của $(L)$ với $C H$. Ta có $H E=\frac{H C+H B-B C}{2}$

Gọi $P$ và $Q$ lần lượt là tiếp điểm của $(I)$ với $A C$ và $B C$ $H D-H E=\frac{H A-A C-H B+B C}{2}=\frac{B C-A C+H A-H B}{2}$ $=\frac{B Q+C Q-A P-C P+H A-H B}{2}=0$ (vì $H$ là tiếp điểm của $(I)$ với $A B$ nên $A H=A P, B H=B Q$

Do đó $D$ trùng $E$. nên hai đường tròn $(J)$ và $(L)$ tiếp xúc nhau

Bài 4. Cho $x, y$ là 2 số thực dương. Tìm giá trị nhỏ nhất của biếu thức:

$P=\frac{16 \sqrt{x y}}{x+y}+\frac{x^{2}+y^{2}}{x y}$

Lời giải.  $P=\frac{16 \sqrt{x y}}{x+y}+\frac{x^{2}+y^{2}}{x y} $

$=\frac{16 \sqrt{x y}}{x+y}+\frac{(x+y)^{2}}{x y}-2 $

$=\frac{8 \sqrt{x y}}{x+y}+\frac{8 \sqrt{x y}}{x+y}+\frac{(x+y)^{2}}{x y}-2 $

$\geq 3 \sqrt[3]{64}-2=10$

Dấu ” $=$ “xảy ra khi và chỉ khi $x=y$ Vậy $P_{\min }=10 \Leftrightarrow x=y$

Bài 5. Cho tam giác $A B C$ có góc $B$ tù. Đường tròn $(O)$ nội tiếp tam giác $A B C$ tiếp xúc với các cạnh $A B, C A, B C$ lần lượt tại $L, H, J$.

a) Các tia $B O, C O$ cắt $L H$ lần lượt tại $M, N$. Chứng minh 4 điểm $B, C, M, N$ cùng thuộc một đường tròn.

b) Gọi $d$ là đường thẳng qua $O$ và vuông góc với $A J ; d$ cắt $A J$ và đường trung trực của cạnh $B C$ lần lượt tại $D$ và $F$. Chứng minh 4 điểm $B, D, F, C$ cùng thuộc một đường tròn.

Lời giải.

a) $N, O$ nằm trên đường trung trực của đoạn $H J$ nên $N H=N J, O H=O J$ $\Rightarrow \triangle N H O=\triangle N J O \Rightarrow \angle N H O=\angle N J O$.

Mà $\angle N H O=\angle N L O$ nên $\angle N J O=\angle N L O \Rightarrow L N O J$ nội tiếp.

Lại có $B L O J$ nội tiếp nên 5 điểm $B, J, O, N, L$ cùng nằm trên một đường tròn.

Suy ra $B N O J$ là tứ giác nội tiếp, suy ra $\angle B N O=90^{\circ}$.

$M, O$ nằm trên đường trung trực của đoạn $L J$ nên $M L=M J, O L=O J \Rightarrow$ $\triangle M O L=\triangle M O J \Rightarrow \angle O L M=\angle O J M$

Mà $\angle O L H=\angle O H L$ nên $\angle O H L=\angle O J M \Rightarrow O H M J$ nội tiếp.

Lại có $O H C J$ nột tiếp nên $O, H, M, C$, $J$ cùng thuộc một đường tròn nên $O M H C$ nội tiếp $\Rightarrow \angle O M C=\angle O H C=90^{\circ}$

$\angle B N C=\angle B M C=90^{\circ} \Rightarrow B M N C$ nội tiếp.

b) Gọi $E$ là giao điểm của $\mathrm{MN}$ và $B C$. Ta chứng minh $O E \perp A J$.

Ta có $O K . O A=O H^{2}=O J^{2}$, suy ra tam giác $O K J$ và $O J A$ đồng dạng, suy ra $\angle O K J=\angle O J A$.

Mặt khác tứ giác $O K E J$ nội tiếp nên $\angle O K J=\angle O E J$.

Do đó $\angle O J A=\angle O E J$, suy ra $O E \perp A J$. Khi đó $O E$ cắt $A J$ tại $D$ và cắt trung trực $B C$ tại $F$.

Xét tam giác $T B C$ chứng minh được $M, N, Q, J$ cùng thuộc đường tròn.

Ta có $E D . E F=E J . E Q=E M . E N=E B . E C$

Suy ra $B D F C$ nội tiếp.

Bài 6. Trên một đường tròn có 9 điểm phân biệt, các điểm này được nối với nhau bởi các đoạn thẳng màu xanh hoặc màu đỏ. Biết rằng mỗi tam giác tạo bởi 3 trong 9 điểm chứa ít nhất một cạnh màu đỏ. Chứng minh rằng tồn tại 4 điểm sao cho 6 đoạn thẳng nối chúng đều có màu đỏ.

Lời giải. Giả sử không tồn tại 4 điểm nào sao cho 6 đoạn thẳng nối chúng đều có màu đỏ.

$-$ Nếu tồn tại một điểm nối ít nhất 4 điểm khác để tạo thành đoạn thẳng màu xanh, giả sử $A$ nối với $B, C, D, E$ tạo thành đoạn màu xanh, khi đó:

$-$ Nếu có hai trong 4 điểm $B, C, D, E$ nối với nhau tạo thành đoạn màu xanh thì mâu thuẫn

$-$ Nếu 4 điểm này đôi một không nối với nhau tạo thành đoạn màu xanh thì cũng mâu thuẫn.

$-$ Nếu mỗi điểm chỉ nối tối đa với 3 điểm khác để tạo thành đoạn màu xanh. Giả sử $A$ nối với $B, C, D$ tạo thành đoạn màu xanh thì $B C, C D, B D$ màu đỏ và còn lại 5 điểm $M, N, P, Q, R$ nối với $A$ tạo thành đoạn màu đỏ.

$-$ Nếu trong 5 điểm đó, điểm nào cũng nối với 2 điểm trong đó tạo thành đoạn màu xanh, khi đó, mỗi điểm chỉ tạo thêm được đoạn màu xanh với 1 điểm nữa khác 5 điểm $M, N, P, Q, R$. Mà 5 điểm này đều phải tạo với một trong 3 điểm $B, C, D$ tạo thành đoạn màu xanh nên có ít nhất hai điểm cùng tạo với 1 điểm trong $B, C, D$ tạo thành đoạn màu xanh, giả sử đoạn $D M$ và $D N$ màu xanh. Khi đó đoạn $M N$ màu đỏ và $M, N$ nối với $B, C$ tạo thành đoạn màu đỏ. Do đó $B, C, M, N$ là 4 điểm khi nối nhau tạo thành 6 đoạn màu đỏ nên mâu thuẫn.

$-$ Nếu trong 5 điểm $M, N, P, Q$, $R$ có 1 điểm nối với 1 trong 4 điểm còn lại tạo thành đoạn màu xanh, giả sử $M N$ màu xanh. Khi đó $P M, Q M, R M$ màu đỏ. Nếu $P Q, Q R, P R$ đều màu xanh thì mâu thuẫn, nếu 1 trong 3 đoạn màu đỏ, giả sử $P Q$ màu đỏ. Khi đó $A, M, P, Q$ là 4 điểm khi nối nhau tạo thành 4 đoạn màu đỏ. (mâu thuẫn).

$-$ Nếu mỗi điểm chỉ nối tối đa với 2 điểm khác tạo thành đoạn màu xanh, giả sử $A B$, $A C$ màu xanh. Khi đó 6 điểm còn lại $M, N, P, Q, R, T$ khi nối $A$ tạo thành màu đỏ. Giả sử $M B$ màu xanh thì $M$ nối được với 1 điểm nữa trong 5 điểm còn lại tạo thành đoạn màu xanh, giả sử $M N$ màu xanh. Khi đó $M P, M Q, M R, M T$ màu đỏ. Trong 4 điểm $P, Q, R, T$ tồn tại 2 điểm nối nhau tạo thành màu đỏ, giả sử $P Q$ màu đỏ. Khi đó $A, M, P, Q$ là 4 điểm nối nhau tạo thành 6 đoạn màu đỏ. Giả sử $M B, M C$ đều màu đỏ thì $M$ tạo được với tối đa 2 điểm nữa trong 5 điểm còn lại thành đoạn màu xanh. Giả sử $M N, M P$ màu xanh thì $M Q, M R, M T$ màu đỏ, trong 3 điểm $Q, R, T$ tồn tại 2 điểm nối nhau tạo thành đoạn màu đỏ, giả sử $Q R$ màu đỏ thì $A, M, Q, R$ là 4 điểm nối nhau tạo thành 6 đoạn màu đỏ (mâu thuẫn).

Vậy tồn tại 4 điểm sao cho 6 đoạn thẳng nối chúng đều có màu đỏ.

 

 

 

 

 

 

 

 

 

 

 

 

 

 

 

 

 

 

 

 

 

 

 

 

 

 

 

 

 

 

 

 

 

 

 

Đề thi và đáp án tuyển sinh vào 10 TPHCM 2020

Đề thi vào lớp 10 TPHCM năm 2020

Bài 1. Cho parabol $ (P): y=\dfrac{1}{4}x^2$ và đường thẳng $ (d): y=-\dfrac{1}{2}x+2 $

a) Vẽ $ (P) $ và $ (d) $ trên cùng hệ trục tọa độ.

b) Tìm tọa độ giao điểm của $ (P) $ và $ (d) $ bằng phép tính.

Giải

a) Bảng giá trị của $(d)$:

Bảng giá trị của $(P)$:

Đồ thị:

b) Phương trình hoành độ giao điểm của $(P)$ và $(d)$:

$\dfrac{1}{4}x^2 = -\dfrac{1}{2}x +2 \Leftrightarrow \dfrac{1}{4}x^2 +\dfrac{1}{2}x-2=0\Leftrightarrow \left[ \begin{array}{l} x=2\Rightarrow y=1\\ x=-4\Rightarrow y=4 \end{array}\right. $

Vậy tọa độ giao điểm $\left( 2;\, 1\right) $, $\left( -4;\, 4\right) $

Bài 2. Cho phương trình: $ 2x^2-5x-3=0 $ có 2 nghiệm $ x_1; x_2 $.

Không giải phương trình, hãy tính giá trị của biểu thức: $ A=(x_1+2x_2)(x_2+2x_1) $.

Giải

Ta có: $\Delta = \left( -5\right) ^2-4\cdot 2\cdot (-3)=49>0\Rightarrow $ Phương trình có hai nghiệm phân biệt.

Áp dụng định lý Viete ta có: $\left\{ \begin{array}{l} S=x_1+x_2=\dfrac{5}{2}\\ P=x_1x_2=-\dfrac{3}{2} \end{array} \right. $

Ta có: $A=\left( x_1+2x_2\right) \left( x_2+2x_1\right)=2\left( x_1^2 +x_2^2\right) +5x_1x_2=2\left( S^2-2P\right) +5P=11 $

Vậy $A=11$

Bài 3. Quy tắc sau đây cho ta biết CAN, CHI của năm X nào đó.

Để xác định CAN, ta tìm số dư $ r $ trong phép chia $X$ cho $10$ và tra vào bảng $1$.

Để xác định CHI, ta tìm số dư $ s $ trong phép chia $X$ cho $12$ và tra vào bảng $2$.

Ví dụ: năm $2020$ có CAN là Canh, có CHI là Tí.

Bảng 1

Bảng 2

a) Em hãy sử dụng quy tắc trên để xác định CAN, CHI của năm $2005$?

b) Bạn Hằng nhớ rằng Nguyễn Huệ lên ngôi hoàng đế, hiệu là Quang Trung vào năm Mậu Thân nhưng không nhớ rõ đó là năm bao nhiêu mà chỉ nhớ là sự kiện trên xảy ra vào cuối thế kỉ $18$. Em hãy giúp Hằng xác định chính xác năm đó là năm bao nhiêu?

Giải

a) Năm $2005$ có CAN là Ất, có CHI là Dậu.

b) Vì năm hoàng đế Nguyễn Huệ lên ngôi là cuối thế kỉ $18$ nên năm đó có dạng $\overline{17ab}$ với $a,\ b\in \mathbb{N}$ và $0\le a,\ b\le 9$

Năm đó có CAN là Mậu nên ta có $\overline{17ab}$ chia $10$ dư $8$ suy ra chữ số tận cùng $b=8$

Năm đó có CHI là Thân nên ta có $\overline{17a8}$ chia hết cho $12$. Suy ra $\overline{17a8}$ chia hết cho $3$.

Khi đó: $1+7+a+8= 16+a\ \vdots \ 3 \Rightarrow a\in \left\{ 2;\ 5;\ 8\right\} $

Với $a=2\Rightarrow 1728$ chia $10$ dư $8$ và $1728$ chia $12$ dư $0$.

Với $a=5\Rightarrow 1758$ chia $10$ dư $8$ và $1758$ chia $12$ dư $6$ (loại).

Với $a=8\Rightarrow 1788$ chia $10$ dư $8$ và $1788$ chia $12$ dư $0$.

Vì sự kiện xảy ra vào cuối thế kỉ $18$ nên năm đó là năm $1788$.

Bài 4. Cước điện thoại $ y $ (nghìn đồng) là số tiền mà người sử dụng điện thoại cần trả hàng tháng, nó phụ thuộc vào lượng thời gian gọi $ x $ (phút) của người đó trong tháng. Mối liên hệ giữa hai đại lượng này là một hàm số bậc nhất $ y=ax+b. $ Hãy tìm $ a,b $ biết rằng nhà bạn Nam trong tháng $5$ đã gọi $100$ phút với số tiền là $40$ nghìn đồng và trong tháng $6$ đã gọi $40$ phút với số tiền $28$ nghìn đồng.

Giải

Với $x=100$ và $y=40$ ta có $40=100a+b$

Với $x=40$ và $y=28$ ta có $28=40a+b$

Ta có hệ phương trình: $\left\{ \begin{array}{l} 100a+b=40\\ 40a+b=28 \end{array}\right. $ $\Leftrightarrow \left\{ \begin{array}{l} a=\dfrac{1}{5}\\ b=20  \end{array}\right. $

Vậy $a=\dfrac{1}{5}$ và $b=20$

Bài 5. Theo quy định của cửa hàng xe máy, để hoàn thành chỉ tiêu trong $1$ tháng, mỗi nhân viên phải bán được trung bình một chiếc xe máy trong một ngày. Nhân viên nào hoàn thành chỉ tiêu trong một tháng thì nhận được lương cơ bản là $8 000 000$ đồng. Nếu trong tháng nhân viên nào bán vượt chỉ tiêu thì được hưởng thêm $8\%$ tiền lời của số xe máy bán vượt chỉ tiêu đó. Trong tháng $5$ (có $31$ ngày), anh Thành nhận được số tiền là $9 800 000$ đồng (bao gồm cả lương cơ bản và tiền thưởng thêm của tháng đó). Hỏi anh Thành đã bán được bao nhiêu chiếc xe máy trong tháng $5$, biết rằng mỗi xe máy bán ra thì cửa hàng thu lời được $2 500 000$ đồng.

Giải

Tháng $5$ có $31$ ngày nên số xe máy tiêu chuẩn phải bán được là $31$ xe.

Gọi $x$ ($x>0$) là số xe máy anh Thành đã bán vượt chỉ tiêu.

Số tiền anh Thành được thưởng thêm là: $8\% \cdot 2\, 500\, 000 \cdot x = 200\, 000x$

Ta có phương trình: $200\, 000x = 9\, 800\, 000 -8\, 000\, 000 \Rightarrow x=9$

Vậy anh Thành đã bán được $40$ xe máy trong tháng $5$.

Bài 6. Anh Minh vừa mới xây một cái hồ trữ nước cạnh nhà có hình dạng hộp chữ nhật có kích thước $2m \times 2 m \times 1 m$. Hiện hồ chưa có nước nên anh Minh phải ra sông lấy nước. Mỗi lần ra sông anh gánh được $1$ đôi nước đầy gồm $2$ thùng hình trụ bằng nhau có bán kính đáy $0,2 \ m$, chiều cao $0,4 \ m$.

a)Tính lượng nước ($m^3$) anh Minh đổ vào hồ sau mỗi lần gánh (ghi kết quả làm tròn đến $2$ chữ số thập phân). Biết trong quá trình gánh nước về thì lượng nước bị hao hụt khoảng $10\%$ và công thức tính thể tích hình trụ là $V = \pi R^2h$.

b) Hỏi anh Minh phải gánh ít nhất bao nhiêu lần để đầy hồ? Bỏ qua thể tích thành hồ.

Giải

a) Thể tích nước anh gánh được trong hai thùng là: $V = 2\cdot \pi R^2h =\dfrac{4\pi }{125}$ ($m^3$)

Lượng nước anh Minh đổ vào hồ sau mỗi lần gánh là: $90\% \cdot V=\dfrac{18\pi }{625}\approx 0,09$ ($m^3$)

b) Thể tích hồ trữ nước là: $V_{\text{hồ}}=2\cdot 2\cdot 1=4$ ($m^3$)

Ta có: $\dfrac{V_{\text{hồ}}}{V}\approx 44,21 $

Vậy anh Minh phải gánh ít nhất $45$ lần để đổ nước đầy hồ.

Bài 7. Sau buổi sinh hoạt ngoại khóa nhóm bạn của Thư rủ nhau đi ăn kem ở một quán gần trường. Do quán mới khai trương nên có khuyến mãi, bắt đầu từ ly thứ $5$ giá mỗi ly kem được giảm $1 500$ đồng so với giá ban đầu. Nhóm của Thư mua $9$ ly kem với số tiền là $154 500$ đồng. Hỏi giá của một ly kem ban đầu?

Giải

Gọi $x$ (đồng) là giá tiền của một ly kem khi chưa giảm (ĐK: $x \geq 1 500$ đồng)

$ \Rightarrow$ Giá tiền ly kem từ ly thứ 5 trở đi là: $x-1 500$ (đồng)

Theo bài ra ta có:  $4.x+5 (x- 1500)= 154 500 \Leftrightarrow x=18 000$ ( đồng)

Vậy giá tiền ly kem ban đầu là: $18000$ đồng.

Bài 8. Cho đường tròn tâm $O$; bán kính $R$ và điểm $A$ nằm ngoài đường tròn sao cho $OA>2R$. Từ $A$ kẻ $2$ tiếp tuyến $AD$; $AE$ đến đường tròn ($O$) ($D$; $E$ là hai tiếp điểm). Lấy điểm $M$ nằm trên cung nhỏ $DE$ sao cho $MD >ME$. Tiếp tuyến của đường tròn (O) tại $M$ cắt $AD$; $AE$ lần lượt tại $I$; $J$. Đường thẳng $DE$ cắt $OJ$ tại $F$ .

a) Chứng minh: $OJ$ là đường trung trực của đoạn thẳng $ME$ và $\angle OMF=\angle OEF$.

b) Chứng minh: tứ giác $ODIM$ nội tiếp và $5$ điểm $I;\ D; \ O;\ F;\ M$ cùng nằm trên một đường tròn.

c) Chứng minh: $\angle JOM=\angle IOA$ và $\sin \angle IOA=\dfrac{MF}{IO}$

Giải

a)

  • Ta có: $ \left\lbrace \begin{array}{l} OM=OE (=R)\\ MJ=EJ \end{array} \right. \Rightarrow OJ$ là đường trung trực của đoạn $ME$
  • Ta có: $OJ$ là tia phân giác của góc $\angle EOM \Rightarrow \angle EOJ =\angle MOJ$

Xét $\triangle EOF $ và $\triangle MOF$ ta có: $OF$ chung, $OM=OE$, $\angle EOF= \angle MOF$

$\Rightarrow \triangle EOF = \triangle MOF \Rightarrow \angle OMF =\angle OEF$.

b)

  • Tứ giác $ODIM$ có: $\angle ODI +\angle OMI =90^{\circ} +90^{\circ}= 180^{\circ}$

$\Rightarrow $ Tứ giác $ODIM$ là tứ giác nội tiếp $(1)$.

  • Ta có: $\angle ODE =\angle OED$ và $\angle OEF =\angle OMF$

$ \Rightarrow \angle ODF =\angle OMF \Rightarrow $ Tứ giác $ODMF$ là tứ giác nội tiếp $(2)$.

Từ $(1)$, $(2)$ ta có: $5$ điểm $I,D,O,F,M$ cùng thuộc một đường tròn.

c)

  • Tứ giác $IDFM$ nội tiếp nên ta có: $\angle IOF =\angle IDF = \dfrac{1}{2}$ sđ cung $IF$ $(3)$

Tứ giác $ADOE$ nội tiếp nên : $\angle ADE =\angle AOE$ $(4)$

Từ $$(3)$, $(4)$ ta có: $\angle IOF =\angle AOE$

Mà ta có: $ \angle IOF =\angle IOA+ \angle AOF$

$ \angle AOE=\angle AOF +\angle EOF$

Suy ra: $ \angle EOF =\angle IOA$

Mặt khác $\angle EOF =\angle JOM$ ( do $OJ$ là tia phân giác$ EOM$ )

Vậy $\angle JOM =\angle IOA$ $(5)$

  • Ta có: $\triangle JMF \backsim \triangle JOI$ (g.g) $\Rightarrow \dfrac{JM}{JO} =\dfrac{MF}{OI}$ $(6)$

Xét tam giác $OMJ$ vuông tại $M$ nên: $\sin \angle JOM =\dfrac{MJ}{OJ}$ (7)

Từ $(5), (6), (7)$ suy ra: $ \sin \angle IOA=\dfrac{MF}{IO}$